Top Banner
Borivali | Andheri |Powai & Pune Contact : 9029077114/ 8097607504 / 9619160261 / 8149774534 www.catking.in Critical Reasoning Questions
72

Critical Reasoning Questions - CATKing Educare · Critical Reasoning Questions . Borivali | Andheri |Powai & Pune Contact : 9029077114/ 8097607504 / 9619160261 / 8149774534 1. Homeowners

Jul 28, 2020

Download

Documents

dariahiddleston
Welcome message from author
This document is posted to help you gain knowledge. Please leave a comment to let me know what you think about it! Share it to your friends and learn new things together.
Transcript
Page 1: Critical Reasoning Questions - CATKing Educare · Critical Reasoning Questions . Borivali | Andheri |Powai & Pune Contact : 9029077114/ 8097607504 / 9619160261 / 8149774534 1. Homeowners

Borivali | Andheri |Powai & Pune

Contact : 9029077114/ 8097607504 / 9619160261 / 8149774534

www.catking.in

Critical Reasoning Questions

Page 2: Critical Reasoning Questions - CATKing Educare · Critical Reasoning Questions . Borivali | Andheri |Powai & Pune Contact : 9029077114/ 8097607504 / 9619160261 / 8149774534 1. Homeowners

Borivali | Andheri |Powai & Pune

Contact : 9029077114/ 8097607504 / 9619160261 / 8149774534

www.catking.in

1. Homeowners aged 40 to 50 are more likely to purchase ice cream and are more likely to purchase it in larger

amounts than are members of any other demographic group. The popular belief that teenagers eat more ice cream

than adults must, therefore, be false.

The argument is flawed primarily because the author

(A) fails to distinguish between purchasing and consuming

(B) does not supply information about homeowners in age groups other than 40 to 50

(C) depends on popular belief rather than on documented research findings

(D) does not specify the precise amount of ice cream purchased by any demographic group

(E) discusses ice cream rather than more nutritious and healthful foods

2. Because no employee wants to be associated with bad news in the eyes of a superior, information about serious

problems at lower levels is progressively softened and distorted as it goes up each step in the management

hierarchy. the chief executive is therefore, less well informed about problems at lower levels than are his or her

subordinates at those levels.

The conclusion drawn above is based on the assumption that

a) problems should be solved at levels in the management hierarchy at which they occur

b) employees should be rewarded for accurately reporting problems to their superiors

c) problems-solving ability is more important at higher levels than it is at lower levels of the management

hierarchy

d) chief executives obtain information about problems at lower levels from the no source other than their

subordinates

e) some employees are more are concerned about truth than about the way they are perceived by their superiors

3. A company is considering changing its policy concerning daily working hours. Currently, this company requires all

employees to arrive at work at 8 a.m. The proposed policy would permit each employee to decide when to arrives

from as early as 6 a.m. to as late as 11 a.m.

The adoption of this policy would be most likely to decrease employees productivity if the employees job

functions required them to

(A) work without interruption from other employees

(B) consult at least once a day with employees from other companies

(C) submit their work for a supervisors eventual approval

(D) interact frequently with each other throughout the entire workday

(E) undertake projects that take several days to complete

4. Manufacturers are now required to make all cigarette lighters child-resistant by equipping them with safety

levers. But this change is unlikely to result in a significant reduction in the number of fi res caused by children

Page 3: Critical Reasoning Questions - CATKing Educare · Critical Reasoning Questions . Borivali | Andheri |Powai & Pune Contact : 9029077114/ 8097607504 / 9619160261 / 8149774534 1. Homeowners

Borivali | Andheri |Powai & Pune

Contact : 9029077114/ 8097607504 / 9619160261 / 8149774534

www.catking.in

playing with lighters, because children given the opportunity can figure out how to work the safety levers and

………..

Which of the following, if true, most logically completes the argument below?

(A) the addition of the safety levers has made lighters more expensive than they were before the requirement was

instituted

(B) adults are more likely to leave child-resistant lighters than non-child-resistant lighters in places that are

accessible to children

(C) many of the fi res started by young children are quickly detected and extinguished by their parents

(D) unlike child-resistant lighters, lighters that are not child-resistant can be operated by children as young as two

years old

(E) approximately 5,000 fi res per year have been attributed to children playing with lighters before the safety

levers were required

5. Burglar alarm systems can be inaccurate in two ways: they can fail to detect an intruder when one is present, or

they can falsely report the presence of an intruder. To detect the presence of an intruder most accurately,

homeowners should purchase the system that is least likely to falsely report the presence of an intruder.

If true, which of the following would most strengthen the above recommendation?

A) The police respond to automatic alerts from burglar alarms more quickly than they do to telephone calls

reporting the presence of an intruder.

B) The burglar alarm system that is least likely to sound its alarm when there is no intruder present is equal in

price to all other major brands of alarm systems.

C) Local police forces vary in their response rates to burglar alarm systems.

D) All burglar alarm systems have the same rate of mistaking an animal for a burglar.

E) All burglar alarm systems have the same rate of failing to detect an intruder when one is present.

6. Studies have shown that elderly people who practice a religion are much more likely to die immediately after an

important religious holiday period than immediately before one. Researchers have concluded that the will to live

can prolong life, at least for short periods of time.

Which of the following, if true, would most strengthen the researchers' conclusion?

a) elderly people who practice a religion are less likely to die immediately before or during an important religious

holiday than at any other time of the year

b) elderly people who practice a religion appear to experience less anxiety at the prospect of dying than do other

people

c) some elderly people who do practice a religion live much longer than most elderly people who do not

d) most elderly people who participate in religious holidays have different reasons for participating than young

people do

Page 4: Critical Reasoning Questions - CATKing Educare · Critical Reasoning Questions . Borivali | Andheri |Powai & Pune Contact : 9029077114/ 8097607504 / 9619160261 / 8149774534 1. Homeowners

Borivali | Andheri |Powai & Pune

Contact : 9029077114/ 8097607504 / 9619160261 / 8149774534

www.catking.in

e) many religious have important holidays in the spring and fall, seasons with the lowest death rates for elderly

people

7. Sania: The newest workers in the workforce are the most effective digital marketing employees because they are

more likely to use social networking websites and tools themselves.

Carlos: But effective digital marketing also requires very technical expertise, such as search engine optimization,

that is best learned on the job via prolonged exposure and instruction.

Carlos responds to Sania by

(A) demonstrating that Sania's conclusion is based upon evidence that is not relevant to the given situation

(B) questioning the accuracy of the evidence presented by Sania in support of her conclusion

(C) reinforcing Sania's argument by contributing an additional piece of evidence in support of her conclusion

(D) pointing out differences in the qualifications desired by different employers seeking digital marketing

employees

(E) providing an additional piece of evidence that undermines a portion of Sania's claim

8. State X recently decided to cut state funding for the public library system in County X. To help counteract this cut

in funding, the county library system has increased library late fees from $.10 per day to $.12 per day. Since the

fee increase, library administrators have seen no decline in the number of overdue books. The director of the

county library system concludes that the fee hike has helped to counteract the cut in state funding. Which of the

following statements, if true, most strengthens the director’s claim?

A. Since the fee increase, library administrators have noted a significant decrease in the number of books

borrowed each day.

B. The library system incurred minor costs to make its visitors aware of the late fee increase.

C. Since the fee increase, there has been no significant change in the average number of days that books are

overdue before they are returned.

D. The library system in County X tracks its books through a very advanced database system, allowing library

administrators to have access to very accurate statistics on the number of overdue books at any given time.

E. Since the reduction in state funding, the library system in County X has eliminated 10% of its staff, creating a

2%reduction in costs.

9. A report on acid rain concluded, “Most forests in Canada are not being damaged by acid rain.” Critics of the report

insist the conclusion be changed to, “Most forests in Canada do not show visible symptoms of damage by acid rain,

such as abnormal loss of leaves, slower rates of growth, or higher mortality.”

Which of the following, if true, provides the best logical justification for the critics’ insistence that the

report’s conclusion be changed?

(A) Some forests in Canada are being damaged by acid rain.

(B) Acid rain could be causing damage for which symptoms have not yet become visible.

Page 5: Critical Reasoning Questions - CATKing Educare · Critical Reasoning Questions . Borivali | Andheri |Powai & Pune Contact : 9029077114/ 8097607504 / 9619160261 / 8149774534 1. Homeowners

Borivali | Andheri |Powai & Pune

Contact : 9029077114/ 8097607504 / 9619160261 / 8149774534

www.catking.in

(C) The report does not compare acid rain damage to Canadian forests with acid rain damage to forests in other

countries.

(D) All forests in Canada have received acid rain during the past fifteen years.

(E) The severity of damage by acid rain differs from forest to forest.

10. A report on acid rain concluded, “Most forests in Canada are not being damaged by acid rain.” Critics of the report

insist the conclusion be changed to, “Most forests in Canada do not show visible symptoms of damage by acid rain,

such as abnormal loss of leaves, slower rates of growth, or higher mortality.”

Which of the following, if true, provides the best logical justification for the critics’ insistence that the

report’s conclusion be changed?

(A) Some forests in Canada are being damaged by acid rain.

(B) Acid rain could be causing damage for which symptoms have not yet become visible.

(C) The report does not compare acid rain damage to Canadian forests with acid rain damage to forests in other

countries.

(D) All forests in Canada have received acid rain during the past fifteen years.

(E) The severity of damage by acid rain differs from forest to forest.

11. On average, the number of speeding tickets issued in County X every year is three times greater than the number of

speeding tickets issued in County Y during the same period. Therefore, the number of people who exceed the speed

limit must be higher in County X than in County Y. Which of the following describes a flaw in the reasoning above?

A. The argument fails to take into account that the speed limit may be different in the two counties.

B. The argument fails to take into account that the number of tickets issued in County X may reflect a lower

proportion of drivers overall in that county.

C. The argument fails to take into account that a single driver can receive more than one ticket in a given year.

D. The argument fails to take into account that residents of County Y may be more law-abiding overall.

E. The argument fails to take into account that residents of County X may not be aware of the speed limit in that

county.

12. The last members of a now-extinct species of a European wild deer called the giant dear lived in Ireland about

16,000 years ago. Prehistoric cave paintings in France depict this animal as having a large hump on its back.

Fossils of this animal, however, do not show any hump. Nevertheless, there is no reason to conclude that the cave

paintings are therefore inaccurate in this regard, since ______.

Which of following most logically completes the argument?

A. some prehistoric cave paintings in France also depict other animals as having a hump

B. fossils of the giant deer are much more common in Ireland than in France

C. animal humps are composed of fatty tissue, which dose not fossilize

D. the cave paintings of the giant deer were painted well before 16,000 years ago

E. only one currently existing species of deer has any anatomical feature that even remotely resembles a hump

Page 6: Critical Reasoning Questions - CATKing Educare · Critical Reasoning Questions . Borivali | Andheri |Powai & Pune Contact : 9029077114/ 8097607504 / 9619160261 / 8149774534 1. Homeowners

Borivali | Andheri |Powai & Pune

Contact : 9029077114/ 8097607504 / 9619160261 / 8149774534

www.catking.in

13. Because it was long thought that few people would watch lengthy televised political messages, most televised

political advertisements, like commercial advertisements, took the form of short messages. Last year, however,

one candidate produced a half-hour-long advertisement.

During the half hour the advertisement was aired, a substantial portion of the viewing public tuned into the

advertisement. Clearly, then, many more people are interested in watching lengthy televised political messages

than was previously thought.

Which of the following is an assumption on which the argument depends?

A. The candidate’s ratings improved significantly as a result of the half-hour-long political advertisement.

B. Political advertisements have become increasingly influential in determining voters’ decisions at the polls.

C. Many people would appreciate the opportunity to become better acquainted with political candidates’ views on

current political issues.

D. Most people who are interested in political issues watch television regularly.

E. Most of the viewers who tuned in to the candidate’s half-hour-long advertisement last year did not change

channels after the first few minutes.

14. Gortland has long been narrowly self-sufficient in both grain and meat. However, as per capita income in Gortland

has risen toward the world average, per capita consumption of meat has also risen toward the world average, and

it takes several pounds of grain to produce one pound of meat. Therefore, since per capita income continues to

rise, whereas domestic grain production will not increase, Gortland will soon have to import either grain or meat

or both.

Which of the following is an assumption on which the argument depends?

A: The total acreage devoted to grain production in Gortland will not decrease substantially.

B: The population of Gortland has remained relatively constant during the country's years of growing prosperity.

C: The per capita consumption of meat in Gortland is roughly the same across all income levels.

D: In Gortland, neither meat nor grain is subject to government price controls.

E: People in Gortland who increase their consumption of meat will not radically decrease their consumption of

grain.

15. Mayor: Migrating shorebirds stop at our beach just to feed on horseshoe-crab eggs, a phenomenon that attracts

tourists. To bring more tourists, the town council plans to undertake a beach reclamation project to double the

area available to crabs for nesting.

Birdwatcher: Without a high density of crabs on a beach, migrating shorebirds will go hungry because shorebirds

only eat eggs that a crab happens to uncover when it is digging its own nest.

Which of the following, if true, would provide the mayor with the strongest counter to the birdwatcher’s

objection?

Page 7: Critical Reasoning Questions - CATKing Educare · Critical Reasoning Questions . Borivali | Andheri |Powai & Pune Contact : 9029077114/ 8097607504 / 9619160261 / 8149774534 1. Homeowners

Borivali | Andheri |Powai & Pune

Contact : 9029077114/ 8097607504 / 9619160261 / 8149774534

www.catking.in

A. Every year a certain percentage of crabs are caught by fishermen as bait for eel traps.

B. Horseshoe crabs are so prolific that given favorable circumstances their numbers increase rapidly.

C. On average, tourists who come to the town in order to watch birds spend more money there than tourists who

come for other purposes.

D. The additional land made available by the reclamation project will give migrating shorebirds more space.

Some of the migrating shorebirds make only one stop during their migration form South America to Canada

16. In parts of South America, vitamin-A deficiency is a serious health problem, especially among children. In one

region, agriculturists are attempting to improve nutrition by encouraging farmers to plant a new variety of sweet

potato called SPK004 that is rich in beta-carotene, which the body converts into vitamin A. The plan has good

chances of success, since sweet potato is a staple of the region’s diet and agriculture, and the varieties currently

grown contain little beta-carotene.

Which of the following, if true, most strongly supports the prediction that the plan will succeed?

A. The growing conditions required by the varieties of sweet potato currently cultivated in the region are

conditions in which SPK004 can flourish.

B. The flesh of SPK004 differs from that of the currently cultivated sweet potatoes in color and texture, so

traditional foods would look somewhat different when prepared from SPK004.

C. There are no other varieties of sweet potato that are significantly richer in beta-carotene than SPK004 is.

D. The varieties of sweet potato currently cultivated in the region contain some important nutrients that are

lacking in SPK004.

E. There are other vegetables currently grown in the region that contain more beta-carotene than the currently

cultivated varieties of sweet potato do

17. The prairie vole, a small North American grassland rodent, breeds year around, and a group of voles living

togather consists primarily of an extended family, often including two or more litters. Voles commonly live in

large groups from late autumn to winter; from spring thru early autumn, however, most voles live in far smaller

groips. The seasonal variation in groups size can probably be explained by a seasonal variation in mortality

among young voles.

Which of the following, if true, provides the strongest support for the explanation above?

A) It is the spring and in the early summer that prairie vole communities generally contain the highest proportion

of young voles.

B) Prairie vole populations vary dramatically in size from year to year.

C) The prairie vole subsists primarily on broad-leaved plants that are abundant only in spring.

D) Winters in prairie vole's habitat are often harsh, with temperatures that drop well below freezing.

E) Snakes, a major predator of young prairie voles, or active only from spring thru early autumn.

Page 8: Critical Reasoning Questions - CATKing Educare · Critical Reasoning Questions . Borivali | Andheri |Powai & Pune Contact : 9029077114/ 8097607504 / 9619160261 / 8149774534 1. Homeowners

Borivali | Andheri |Powai & Pune

Contact : 9029077114/ 8097607504 / 9619160261 / 8149774534

www.catking.in

18. Extinction is a process that can depend on a variety of ecological, geographical, and physiological variables. These

variables affect different species of organisms in different ways, and should, therefore, yield a random pattern of

extinctions. However, the fossil record shows that extinction occurs in a surprisingly definite pattern, with many

species vanishing at the same time.

Which of the following, if true, forms the best basis for at least a partial explanation of the patterned extinctions

revealed by the fossil record?

(A) Major episodes of extinction can result from widespread environmental disturbances that affect numerous

different species.

(B) Certain extinction episodes selectively affect organisms with particular sets of characteristics unique to their

species.

(C) Some species become extinct because of accumulated gradual changes in their local environments.

(D) In geologically recent times, for which there is no fossil record, human intervention has changed the pattern of

extinctions.

(E) Species that are widely dispersed are the least likely to become extinct

19. Most household appliances use electricity only when in use. Many microwave ovens, however, have built-in clocks

and so use some electricity even when they are not in Use. The clocks each consume about 45 kilowatt-hours per

year. Therefore, house holds whose microwave oven has no built-in clock use 45 kilowatt-hours per year less ,on

average, than do comparable households microwave oven is otherwise similar but has a built-in clock.

Which of the following is an assumption on which the argument depends?

A. Households that do not have a microwave oven use less energy per year, on average, than do households that

have a microwave oven.

B. Microwave ovens with a built-in clock do not generally cost more to buy than microwave ovens without a built-

in clock.

C. All households that have a microwave oven also have either a gas oven or a Conventional electric oven.

D. Households whose microwave oven does not have a built-in clock are no more likely to have a separate electric

clock plugged in than households whose microwave oven has one.

E. There are more households that have a microwave oven with a built-in clock than there are households that

have a microwave oven without a built-in clock.

20. Recent studies show that people between the ages of 13 and 55 produce 65 pounds more garbage per year than

they did in 1995. This increase has led to a higher percentage of the total garbage produced by all the age groups.

This age group constitutes a growing percentage of the population, so it partially explains this rise.

Which of the following can be concluded from passage?

A. People over the age of 55 produce more garbage than people less than the age of 13.

B. Population has risen since 1995.

Page 9: Critical Reasoning Questions - CATKing Educare · Critical Reasoning Questions . Borivali | Andheri |Powai & Pune Contact : 9029077114/ 8097607504 / 9619160261 / 8149774534 1. Homeowners

Borivali | Andheri |Powai & Pune

Contact : 9029077114/ 8097607504 / 9619160261 / 8149774534

www.catking.in

C. People between the age of 13 and 55 are more than half of current population.

D. Before 1995, people below age 13 and above age 55 produced higher percentage of total garbage than they do

now.

E. People between the age of 13 and 55 produce more garbage than those below 13 or older than 55.

21. Early in the twentieth century, Lake Konfa became very polluted. Recently fish populations have recovered as

release of industrial pollutants has declined and the lak waters have become cleaner. Fears are now being voiced

that the planned construction of an oil pipeline across the lake’s bottom might revive pollution and cause the fish

population to decline again. However, a technology for preventing leaks is being installed. Therefore, provided

this technology is effective, those fears are groundless.

The argument depends on assuming which of the following?

A. Apart from development related to the pipeline, there will be no new industria development around the lake

that will create renewed pollution in its waters.

B. Other than the possibility of a leak, there is no realistic pollution threat posed to the lake by the pipeline’s

construction.

C. There is no reason to believe that the leak-preventing technology would be ineffective when installed in the

pipeline in Lake Konfa.

D. Damage to the lake’s fish populations would be the only harm that a leak of oil from the pipeline would cause.

E. The species of fish that are present in Lake Konfa now are the same as those that were in the lake before it was

affected by pollution.

22. Some geologists argue that if oil is as common in unsampled areas of the world as it is in those already sampled,

our current estimate of reserves that exist underground must be multiplied by a factor of 10,000. From this we

can conclude that we can meet the oil needs of the entire world for at least five centuries, even assuming that

future consumption grows at an accelerating rate.

To reach the stated conclusion, the author must assume which of the following?

(A) It is possible to recover the oil contained in unexplored areas of the world

(B) The consumption rate for oil will not grow rapidly

(C) Oil will remain an important energy source for at least 500 years

(D) The world will achieve and maintain zero population growth

(E) New technology will make oil discovery and drilling more feasible than ever before

23. Commentator: The theory of trade retaliation states that countries closed out of any of another country’s markets

should close some of their own markets to the other country in order to pressure the other country to reopen its

markets. If every country acted according to this theory, no country would trade with any other.

The commentator’s argument relies on which of the following assumptions?

Page 10: Critical Reasoning Questions - CATKing Educare · Critical Reasoning Questions . Borivali | Andheri |Powai & Pune Contact : 9029077114/ 8097607504 / 9619160261 / 8149774534 1. Homeowners

Borivali | Andheri |Powai & Pune

Contact : 9029077114/ 8097607504 / 9619160261 / 8149774534

www.catking.in

(A) No country actually acts according to the theory of trade retaliation.

(B) No country should block any of its markets to foreign trade.

(C) Trade disputes should be settled by international tribunal.

(D) For any two countries, at least one has some market closed to the other.

(E) Countries close their markets to foreigners to protect domestic producers.

24. In the past the country of Siduria has relied heavily on imported oil. Siduria recently implemented a program to

convert heating systems from oil to natural gas. Siduria already produces more natural gas each year than it

burns, and oil production in Sidurian oil fields is increasing at a steady pace. If these trends in fuel production and

usage continue, therefore, Sidurian reliance on foreign sources for fuel should decline soon.

Which of the following is an assumption on which the argument depends?

A. In Siduria the rate of fuel consumption is rising no more quickly than the rate of fuel production.

B. Domestic production of natural gas is rising faster than is domestic production of oil in Siduria.

C. No fuel other than natural gas is expected to be used as a replacement for oil in Siduria.

D. Buildings cannot be heated by solar energy rather than by oil or natural gas.

E. All new homes that are being built will have natural-gas-burning heating systems.

25. According to a recent magazine article, of those office employees who typically work 8 hours at the office each day

but sometimes say that they will work at home on a particular day, 25 percent actually work less than one hour.

At the same time, over 90 percent of those same office employees believe they are more productive working at

home than working in their office.

The statements above, if true, best support which of the following conclusions about the office employees

discussed in the article?

a On average, the office employees working at home for a day work fewer hours than office employees working at

the office.

b 10 percent of the office employees are less productive working from home than working in their office.

c At least 15 percent of the office employees do not define productivity exclusively in terms of the number of

hours worked.

d At least 25 percent of the office employees can complete the same amount of work in one hour at home as in 8

hours at the office.

e Some of the office employees make statements regarding their productivity that are not in fact true.

26. Company X experienced a significant loss of market share over a number of years. To strengthen its market

position, the company decided two years ago to increase the percentage of total revenue devoted to research and

development. The following year, the company's market share increased by 10 percent. In response, company

management further increased the amount of money devoted to research and development.

Which of the following investigations is most likely to yield significant information that would help evaluate

Page 11: Critical Reasoning Questions - CATKing Educare · Critical Reasoning Questions . Borivali | Andheri |Powai & Pune Contact : 9029077114/ 8097607504 / 9619160261 / 8149774534 1. Homeowners

Borivali | Andheri |Powai & Pune

Contact : 9029077114/ 8097607504 / 9619160261 / 8149774534

www.catking.in

whether the company's decision to further increase the research and development budget was warranted?

A. Determining how many new products the company has created over the past year

B. Determining the percentage of revenue that other companies devote to research and development

C. Determining whether any of the company's competitors have withdrawn from the market in the past two years

D. Determining the maximum percentage of revenue that the company can devote to research and development

without affecting the company's production budget

E. Determining whether all of the company's new products have fared equally well in the marketplace

27. The popular notion that a tree’s age can be determined by counting the number of internal rings in its trunk is

generally true. However, to help regulate the internal temperature of the tree, the outermost layers of wood

of the Brazilian ash often peel away when the temperature exceeds 95 degrees Fahrenheit, leaving the tree with

fewer rings than it would otherwise have. So only if the temperature in the Brazilian ash’s environment never

exceeds 95 degrees Fahrenheit will its rings be a reliable measure of the tree’s age.

Which of the following is an assumption on which the argument

above depends?

A. The growth of new rings in a tree is not a function of levels of precipitation.

B. Only the Brazilian ash loses rings because of excessive heat.

C. Only one day of temperatures above 95 degrees Fahrenheit is needed to

cause the Brazilian ash to lose a ring.

D. The internal rings of all trees are of uniform thickness.

E. The number of rings that will be lost when the temperature exceeds 95

degrees Fahrenheit is not predictable.

28. In an effort to reduce the number of deer, and therefore decrease the number of automobile accidents caused by

deer, the government lengthened the deer hunting season earlier this year. Surprisingly, the number of accidents

caused by deer has increased substantially since the introduction of the longer hunting season.

All of the following, if true, help to explain the increase in traffic accidents caused

by deer EXCEPT:

(A) The presence of humans in the woods causes the deer to move to new areas, which causes the deer to cross

roads more frequently than normal.

(B) In the area where the deer live, traffic has increased substantially precisely because of the lengthened hunting

season.

(C) Most automobile accidents involving deer result from cars swerving to avoid deer, and leave the deer in

question unharmed.

(D) Deer tend to bolt when hearing gunshots or other loud sounds and are more likely to run across a road

without warning.

Page 12: Critical Reasoning Questions - CATKing Educare · Critical Reasoning Questions . Borivali | Andheri |Powai & Pune Contact : 9029077114/ 8097607504 / 9619160261 / 8149774534 1. Homeowners

Borivali | Andheri |Powai & Pune

Contact : 9029077114/ 8097607504 / 9619160261 / 8149774534

www.catking.in

(E) A new highway was recently built directly through the state's largest forest, which is the primary habitat of the

state's deer population.

29. In the earliest stages of the common law, a party could have a case heard by a judge only upon the payment of a

fee to the court, and then only if the case fit within one of the forms for which there existed a writ. At first the

number of such formalized cases of action was very small, but judges invented new forms which brought more

cases and greater revenues.

Which of the following conclusions is most strongly suggested by the paragraph above?

(A) Early judges often decided cases in an arbitrary and haphazard manner.

(B) In most early cases, the plaintiff rather than the defendant prevailed.

(C) The judiciary at first had greater power than either the legislature or the executive.

(D) One of the motivating forces for the early expansion in judicial power was economic considerations.

(E) The first common law decisions were inconsistent with one another and did not form a coherent body of law.

30. A proposed ordinance requires the installation in new homes of sprinklers automatically triggered by the

presence of a fire. However, a home builder argued that because more than 90 percent of residential fires are

extinguished by a household member, residential sprinklers would only marginally decrease property damage

caused by residential fires.

Which of the following, if true, would most seriously weaken the home builder’s argument?

(A) Most individuals have no formal training in how to extinguish fires.

(B) Since new homes are only a tiny percentage of available housing in the city, the new ordinance would be

extremely narrow in scope.

(C) The installation of smoke detectors in new residences costs significantly less than the installation of

sprinklers.

(D) In the city where the ordinance was proposed, the average time required by the fire department to respond to

a fire was less than the national average.

(E) The largest proportion of property damage that

results from residential fires is caused by fires that start when no household member is present.

31. A major health insurance company in Lagolia pays for special procedures prescribed by physicians only if the

procedure is first approved as “medically necessary” by a company-appointed review panel. The rule is intended

to save the company the money it might otherwise spend on medically unnecessary procedures. The company has

recently announced that in order to reduce its costs, it will abandon this rule.

Which of the following, if true, provides the strongest justification for the company’s decision?

(A) Patients often register dissatisfaction with physicians who prescribe nothing for their ailments.

(B) Physicians often prescribe special procedures that are helpful but not altogether necessary for the health of

Page 13: Critical Reasoning Questions - CATKing Educare · Critical Reasoning Questions . Borivali | Andheri |Powai & Pune Contact : 9029077114/ 8097607504 / 9619160261 / 8149774534 1. Homeowners

Borivali | Andheri |Powai & Pune

Contact : 9029077114/ 8097607504 / 9619160261 / 8149774534

www.catking.in

the patient.

(C) The review process is expensive and practically always results in approval of the prescribed procedure.

(D) The company’s review process does not interfere with the prerogative of physicians, in cases where more

than one effective procedure is available, to select the one they personally prefer.

(E) The number of members of the company-appointed review panel who review a given procedure depends on

the cost of the procedure.

32. CEO: Over the past several years, we have more than doubled our revenues but profits have steadily declined

because an increasing number of customers have failed to pay their balances. In order to compensate for these

higher default rates, we will increase the interest charged on outstanding balances from an annual percentage

rate (APR) of 9.5% to an APR of 12%. This increase will be sufficient to compensate for the current rate of defaults

and allow us to increase our profits.

Which of the following statements, if true, would most seriously undermine a plan

to increase interest rates in order to spur profitable growth?

(A) Many other companies have experienced a similar trend in their default rates.

(B) The company's operating expenses are above the industry average and can be substantially reduced, thus

increasing margins.

(C) The increase in default rates was due to a rise in unemployment, but unemployment rates are expected to

drop in the coming months.

(D) The proposed increase in the APR will, alone, more than double the company's profit margins.

(E) An increase in the APR charged on credit card balances often results in higher rates of default.

33. Many banks have a drive-through automated teller machine, built into the wall flanking the driver's side of a

narrow aisle. Even though blind people obviously do not drive, these machines nonetheless invariably feature

Braille – the system of raised dots used by the blind for reading and writing – in addition to the standard letters

and numerals on their control panels.

Which of the following, if true, does the most to explain the situation described?

a)In most automobiles, the rear windows on the driver's side only roll down about halfway.

b)Moderately visually impaired people who are still allowed to drive typically do not use Braille to read or write.

c)Blind people can often read Braille just as quickly and accurately as sighted people can read regular writing.

d)The embossing technique used to form the raised dots does not typically add much to the overall cost of the

automated teller machine.

e)In order to maximize profits per unit, manufacturers produce only a single model of automated teller machine.

34. Plan: Concerned about the welfare of its senior citizens, the government of Runagia decided two years ago to

increase by 20 percent the government-provided pension paid to all Runagians age sixty-five and older.

Result: Many Runagian senior citizens are no better off financially now than they were before the increase.

Page 14: Critical Reasoning Questions - CATKing Educare · Critical Reasoning Questions . Borivali | Andheri |Powai & Pune Contact : 9029077114/ 8097607504 / 9619160261 / 8149774534 1. Homeowners

Borivali | Andheri |Powai & Pune

Contact : 9029077114/ 8097607504 / 9619160261 / 8149774534

www.catking.in

Further information: The annual rate of inflation since the pension increase has been below 5 percent, and

the increased pension has been duly received by all eligible Runagians.

In light of the further information, which of the following, if true, does most to explain the result that

followed implementation of the plan?

(A) The majority of senior citizens whose financial position has not improved rely entirely on the government

pension for their income.

(B) The Runagian banking system is so inefficient that cashing a pension check can take as much as three weeks.

(C) The prices of goods and services that meet the special needs of many senior citizens have increased at a rate

much higher than the rate of inflation.

(D) The pension increase occurred at a time when the number of Runagians age sixty-five and older who were

living below the poverty level was at an all-time high.

(E) The most recent pension increase was only the second such increase in the last ten years.

35. The public often protests when an unregulated service industry is found to be corrupt.

However, regulation often leads to increased costs for the consumer. Fewer companies

survive in a regulated market, leading to decreased competition and higher prices. The public

then responds negatively to the increased costs of these services. The statements above best

support which of the following?

A) Service industries should not be regulated.

b) The public should not protest unregulated services.

c) Only unregulated services are subject to public protest.

d) The public is sometimes the cause of its own complaints.

e) Decreased competition always leads to increased prices

36. People tend to estimate the likelihood of an event's occurrence according to its salience; that is, according to how

strongly and how often it comes to their attention. By placement and headlines, newspapers emphasize stories

about local crime over stories about crime elsewhere and about many other major events.

It can be concluded on the basis of the statements above that, if they are true, which of the following is most

probably also true?

A. The language used in newspaper headlines about local crime is inflammatory and fails to respect the rights of

suspects.

B. The coverage of international events in newspapers is neglected in favor of the coverage of local events.

C. Readers of local news in newspapers tend to overestimate the amount of crime in their own localities relative to

the amount of crime in other places.

D. None of the events concerning other people that are reported in newspapers is so salient in people's minds as

Page 15: Critical Reasoning Questions - CATKing Educare · Critical Reasoning Questions . Borivali | Andheri |Powai & Pune Contact : 9029077114/ 8097607504 / 9619160261 / 8149774534 1. Homeowners

Borivali | Andheri |Powai & Pune

Contact : 9029077114/ 8097607504 / 9619160261 / 8149774534

www.catking.in

their own personal experiences.

E. The press is the news medium that focuses people's attention most strongly on local crimes.

37. The Eurasian ruffe, a fish species inadvertently introduced into North America’s Great Lakes in recent years,

feeds on the eggs of lake whitefish, a native species, thus threatening the lakes’ natural ecosystem. To help

track the ruffe’s spread, government agencies have produced wallet-sized cards about the ruffe. The cards

contain pictures of the ruffe and explain the danger they pose; the cards also request anglers to report any ruffe

they catch.

Which of the following, if true, would provide most support for the prediction that the agencies’ action will have

its intended effect?

(A) The ruffe has spiny fins that make it unattractive as prey.

(B) Ruffe generally feed at night, but most recreational fi shing on the Great Lakes is done during daytime hours.

(C) Most people who fi sh recreationally on the Great Lakes are interested in the preservation of the lake

whitefi sh because it is a highly prized game fish.

(D) The ruffe is one of several nonnative species in the Great Lakes whose existence threatens the survival of

lake whitefish populations there.

(E) The bait that most people use when fi shing for whitefi sh on the Great Lakes is not attractive to ruffe.

38. Crowding on Mooreville’s subway frequently leads to delays, because it is difficult for passengers to exit from the

trains. Subway ridership is projected to increase by 20 percent over the next 10 years. The Metroville Transit

Authority plans to increase the number of daily train trips by only 5 percent over the same period. Officials

predict that this increase is sufficient to ensure that the incidence of delays due to crowding does not increase.

Which of the following, if true, provides the strongest grounds for the officials’ prediction?

A. By changing maintenance schedules, the Transit Authority can achieve the 5 percent increase in train trips

without purchasing any new subway cars.

B. The Transit Authority also plans a 5 percent increase in the number of bus trips on routes that connect to

subways.

C. For most commuters who use the subway system, there is no practical alternative public transportation

available.

D. Most of the projected increase in ridership is expected to occur in off-peak hours when trains are now sparsely

used.

E. The 5 percent increase in the number of train trips can be achieved without an equal increase in Transit

Authority operational costs.

39. As many as 98,000 people die each year due to medical error. In a campaign to reduce lethal errors, thousands of

hospitals introduced six key changes, including rapid-response teams, re-checks of patient medication, and new

guidelines for preventing infection. The campaign estimated that, over an 18-month period, more than 100,000

Page 16: Critical Reasoning Questions - CATKing Educare · Critical Reasoning Questions . Borivali | Andheri |Powai & Pune Contact : 9029077114/ 8097607504 / 9619160261 / 8149774534 1. Homeowners

Borivali | Andheri |Powai & Pune

Contact : 9029077114/ 8097607504 / 9619160261 / 8149774534

www.catking.in

lives were saved as a direct result of the program.

Which of the following can be most properly inferred from the above statements?

A) Doctors and nurses should be more careful when doing their jobs.

B) The campaign saved all of the people who otherwise would have died due to medical error in that time period.

C) In the future, no one will die because of medical error.

D) If the campaign had not been implemented, more than 100,000 people might have died during the 18-month

period due to medical error.

E) The key changes initiated by the campaign will continue to be implemented in the future.

40. Alan: Industrialisation helps in achieving a variety of social objectives such as employment, gender equality and

poverty eradication. Thus, Industrialization must be promoted extensively for the welfare of a nation.

Charlie: But you must also consider that extensive industrial processes can have negative environmental impacts,

causing climate change, loss of natural resources and extinction of species. These threaten the economic and

social welfare of a nation. Thus, the nation should promote the positive impacts of industrial development while

limiting or eliminating its negative impacts.

Charlie objects to Alan’s argument by

A. showing that Alan has based his argument on limited understanding of the impact of Industrialisation

B. claiming that Alan has exaggerated the positive impacts of Industrialisation.

C. Rejecting Alan’s argument as flawed

D. contradicting the evidence that Alan has used to support his argument.

E. suggesting that negative impacts of Industrialisation offset its positive impacts.

41. The tulu, a popular ornamental plant, does not reproduce naturally, and is only bred and sold by specialized

horticultural companies. Unfortunately, the tulu is easily devastated by a contagious fungal rot. The govt ministry

plans to reassure worried gardeners by requiring all tulu plants to be tested for fungal rot before being sold.

However, infected plats less than 30 weeks old have generally not built enough fungal rot to be detected relaibly.

And many tulu plants are sold before they are 24 weeks old.

which of the following, if performed by the govt ministry, could logically be expected to overcome the problem

with their plan to test fungal rot.

A) Releasing a general announcement that tulu plants less than 30 weeks old cannot be effectively tested for

fungal rot

B) Requiring all tulu plants less than 30 weeks old to be labeled as such.

C) researching possible ways to test tulu plants less than 24 weeks old for fungal rot.

D) Ensuring that tulu plants are not sold before they are 30 weeks old.

E) Quarantining all tulu plants from horticultural companies at which any case of fungal rot has been detected

until those tulu plants can be tested for fungal rot.

Page 17: Critical Reasoning Questions - CATKing Educare · Critical Reasoning Questions . Borivali | Andheri |Powai & Pune Contact : 9029077114/ 8097607504 / 9619160261 / 8149774534 1. Homeowners

Borivali | Andheri |Powai & Pune

Contact : 9029077114/ 8097607504 / 9619160261 / 8149774534

www.catking.in

42. CarStore's sales personnel have an average of fifteen years' experience selling automobiles, and they regularly sell

more cars than other local dealers. Despite this, CarStore has recently implemented a mandatory training

program for all sales personnel.

Which of the following, if true, best explains the facts given above?

A) The sales personnel in CarStore have historically specialized in aggressively selling automobiles and add-on

features.

B) Salespeople at other local dealers average 10 years' experience.

C) It is common for new or less experienced employees to participate in training programs.

D) Pricing information, which used to be confidential, has recently been released on the internet, and many

customers try to negotiate lower prices using this data.

E) Several retailers that compete directly with CarStore use "customer-centered" sales approaches.

43. Joe: Many large, intricate, and precise geometric patterns called “crop circles” consisting of flattened plants in a

field of corn or other grains have mysteriously appeared from time to time on farms in both the United States and

the United Kingdom. The appearance of these crop circles is usually preceded by strange lights in the sky and no

human has ever been caught in the act of creating one of these patterns. These crop circles must be the product of

aliens from outer space.

Steve: Maybe they are a means of communication for some weird secret society. Or perhaps they are all part of an

elaborate hoax by college fraternities. In any case, I seriously doubt that these crop circles are the product of

extra-terrestrial beings.

Steve responds to Joe’s conclusion by _____

a) suggesting alternatives that refute Joe’s conclusion

b) assuming that Joe’s conclusion cannot be correct

c) negating the validity of a premise upon which Joe relies in drawing his conclusion

d) offering alternative explanations consistent with the evidence that Joe cites

e) attacking Joe's conclusion as inconsistent with the available evidence

44. The exchange rate between the currency of Country X and that of Country Y has historically favored the currency

of Country Y. Because of this, citizens of Country Y often take their vacations in Country X, where the exchange

rate makes hotels and restaurants more affordable. Yet, citizens of Country Y rarely purchase clothing or

electronics in Country X, despite the fact that those items are more expensive in their home country, even when

sales taxes are taken into account. Which of the following, if true, would best explain the buying habits of the

citizens of Country Y?

A. Citizens of Country Y prefer the fashions available in their own country.

Page 18: Critical Reasoning Questions - CATKing Educare · Critical Reasoning Questions . Borivali | Andheri |Powai & Pune Contact : 9029077114/ 8097607504 / 9619160261 / 8149774534 1. Homeowners

Borivali | Andheri |Powai & Pune

Contact : 9029077114/ 8097607504 / 9619160261 / 8149774534

www.catking.in

B. Stores in Country X receive the latest fashions and technology several months after they are available in

Country Y.

C. The citizens of Country X resent the buying power of the currency of Country Y.

D. The government of Country Y imposes tariffs on imported goods.

E. The currencies of Country X and Country Y are both weak compared to the currency of Country Z.

45. The Farmsley Film and Performing Arts Center was built three years ago in downtown Metropolis. A recent study

shows that, on average, a person who attends a show at the Farmsley Center spends $96 at other downtown

businesses on the day of the show. This fact, however, does not necessarily indicate that the Farmsley Center is a

significant driver of the economic revitalization of downtown Metropolis, since________________ .

(A) people who do not attend a Farmsley Center show spend $63 on average when shopping in the downtown

area

(B) restaurants near the Farmsley Center tend to be more expensive than restaurants in outlying areas

(C) the Farmsley center generally earns more from films than from plays or other performance art projects

(D) the Farmsley Center is the only downtown theatre large enough to afford to show newly-released major

Hollywood films

(E) most of the people who attend films or performances at the Farmsley Center do so because they are already in

the area to shop

46. In Eastland, from 2000 to 2005, the total consumption of fish increased by 4.5 percent, and the total consumption

of poultry products increased by 9.0 percent. During this time, the population of Eastland increased by 6 percent,

in part due to new arrivals from surrounding areas.

Which of the following, if true, can one infer based on the statements above?

A)For new arrivals to Eastland between 2000 and 2005, fish was less likely to be a major part of families’ diet than

was poultry.

B)In 2005, the residents of Eastland consumed twice as much poultry as fish.

C)The per capita consumption of poultry in Eastland was higher in 2005 than it was in 2000.

D)Between 2000 and 2005, both fish and poultry products were a regular part of the diet of a significant

proportion of Eastland residents.

E)Between 2000 and 2005, the profits of wholesale distributors of poultry products increased at a greater rate

than did the profits of wholesale distributors of fish.

47. Marketing director: If we make our jigsaw puzzles easier to complete then people will not take as long to do each

puzzle and will, therefore, buy more of our puzzles. Additionally, easier puzzles will make people feel more

intelligent and accomplished, without the frustration that often accompanies more difficult puzzles. Clearly,

making our jigsaw puzzles easier to complete is the best way to increase our sales.

Which of the following, if true, would most weaken the marketing director’s argument?

(A) People rarely do puzzles a second time and may give away puzzles after completing them.

(B) People who do jigsaw puzzles also commonly complete difficult crossword puzzles.

Page 19: Critical Reasoning Questions - CATKing Educare · Critical Reasoning Questions . Borivali | Andheri |Powai & Pune Contact : 9029077114/ 8097607504 / 9619160261 / 8149774534 1. Homeowners

Borivali | Andheri |Powai & Pune

Contact : 9029077114/ 8097607504 / 9619160261 / 8149774534

www.catking.in

(C) One of the main reasons that people purchase jigsaw puzzles is for the challenge.

(D) Most jigsaw puzzles are purchased based on the image featured on the completed puzzle.

(E) Easier puzzles may help to expand the market for jigsaw puzzles to new customers.

48. Surveys consistently show that the best-selling ice cream flavor is vanilla, although those who prefer chocolate

rarely order vanilla. Vanilla-flavored candy, then, probably sells better than chocolate-flavored candy. Which of

the following is an assumption upon which the author of the argument relies?

• Because someone prefers vanilla ice cream does not mean he prefers vanilla-flavored candy.

• Children who prefer vanilla ice cream also tend to like chocolate candy.

• Those who prefer neither vanilla nor chocolate ice cream also prefer other flavors of candy.

• Someone who prefers vanilla ice cream may still order chocolate on occasion.

• Preferences for certain ice cream flavors are similar to preferences for candy flavors.

49. A conservation group in the United States is trying to change the long-standing image of bats as frightening

creatures. The group contends that bats are feared and persecuted solely because they are shy animals that are

active only at night.

Which of the following, if true, would cast the most serious doubt on the accuracy of the group’s contention?

(A) Bats are steadily losing natural roosting places such as caves and hollow trees and are thus turning to more

developed areas for roosting.

(B) Bats are the chief consumers of nocturnal insects and thus can help make their hunting territory more

pleasant for humans.

(C) Bats are regarded as frightening creatures not only in the United States but also in Europe, Africa, and South

America.

(D) Raccoons and owls are shy and active only at night; yet they are not generally feared and persecuted.

(E) People know more about the behavior of other greatly feared animal species, such as lions, alligators, and

snakes, than they do about the behavior of bats.

50. A drug that is highly effective in treating many types of infection can, at present, be obtained only from the bark of

the ibora, a tree that is quite rare in the wild. It takes the bark of 5,000 trees to make one kilogram of the drug. It

follows, therefore, that continued production of the drug must inevitably lead to the ibora extinction.

Which of the following, if true, most seriously weakens the argument above?

(A) The drug made from ibora bark is dispensed to doctors from a central authority.

(B) The drug made from ibora bark is expensive to produce.

(C) The leaves of the ibora are used in a number of medical products.

(D) The ibora can be propagated from cuttings and grown under cultivation.

(E) The ibora generally grows in largely inaccessible places.

Page 20: Critical Reasoning Questions - CATKing Educare · Critical Reasoning Questions . Borivali | Andheri |Powai & Pune Contact : 9029077114/ 8097607504 / 9619160261 / 8149774534 1. Homeowners

Borivali | Andheri |Powai & Pune

Contact : 9029077114/ 8097607504 / 9619160261 / 8149774534

www.catking.in

51. The interview is an essential part of a successful hiring program because, with it, job applicants who have

personalities that are unsuited to the requirements of the job will be eliminated from consideration.

The argument above logically depends on which of the following assumptions?

(A) A hiring program will be successful if it includes interviews.

(B) The interview is a more important part of a successful hiring program than is the development of a job

description.

(C) Interviewers can accurately identify applicants whose personalities are unsuited to the requirements of the

job.

(D) The only purpose of an interview is to evaluate whether job applicants’ personalities are suited to the

requirements of the job.

(E) The fit of job applicants’ personalities to the requirements of the job was once the most important factor in

making hiring decisions

52. When three Everett-owned Lightning-built airplanes crashes in the same month, the Everett company

ordered three new Lightning-built airplanes as replacements. This decision surprised many in the airline industry

because, ordinarily when a product is involved in accidents, users become reluctant to buy that product.

Which of the following, if true, provides the best indication that the Everett company’s decision was logically well

supported?

(A) Although during the previous year only one Lightning-built airplane crashed, competing manufacturers had a

perfect safety record.

(B) The Lightning-built airplanes crashed due to pilot error, but because of the excellent quality of the planes

there were many survivors.

(C) The Federal Aviation Association issued new guidelines for airlines in order to standardize safety

requirements governing preflight inspections.

(D) Consumer advocates pressured two major airlines into purchasing safer airplanes so that the public would

be safer while flying.

(E) Many Lightning Airplane Company employees had to be replaced because they found jobs with the

competition.

53. The last members of a now-extinct species of a European wild deer called the giant dear lived in Ireland about

16,000 years ago. Prehistoric cave paintings in France depict this animal as having a large hump on its back.

Fossils of this animal, however, do not show any hump. Nevertheless, there is no reason to conclude that the cave

paintings are therefore inaccurate in this regard, since ______.

Which of following most logically completes the argument?

A. some prehistoric cave paintings in France also depict other animals as having a hump

B. fossils of the giant deer are much more common in Ireland than in France

C. animal humps are composed of fatty tissue, which dose not fossilize

Page 21: Critical Reasoning Questions - CATKing Educare · Critical Reasoning Questions . Borivali | Andheri |Powai & Pune Contact : 9029077114/ 8097607504 / 9619160261 / 8149774534 1. Homeowners

Borivali | Andheri |Powai & Pune

Contact : 9029077114/ 8097607504 / 9619160261 / 8149774534

www.catking.in

D. the cave paintings of the giant deer were painted well before 16,000 years ago

E. only one currently existing species of deer has any anatomical feature that even remotely resembles a hump

54. Physician: The hormone melatonin has shown promise as a medication for sleep disorders when taken in

synthesized form. Because the long-term side effects of synthetic melatonin are unknown, however, I cannot

recommend its use at this time.

Patient: Your position is inconsistent with your usual practice. You prescribe many medications that you know

have serious side effects, so concern about side effects cannot be the real reason you will not prescribe melatonin.

The patient's argument is flawed because it fails to consider that

(A) the side effects of synthetic melatonin might be different from those of naturally produced melatonin

(B) it is possible that the physician does not believe that melatonin has been conclusively shown to be effective

(C) sleep disorders, if left untreated, might lead to serious medical complications

(D) the side effects of a medication can take some time to manifest themselves

(E) known risks can be weighed against known benefits, but unknown risks cannot

55. When feeding, aquatic birds known as phalaropes often spin rapidly on the water’s surface, pecking for food

during each revolution. To execute these spins, phalaropes kick one leg harder than the other. This action creates

upwelling currents. Because plankton on which phalaropes feed typically occurs in greater quantities well below

the surface, it is hypothesized that by spinning phalaropes gain access to food that would otherwise be beyond

their reach.

Which of the following, if true, most strongly supports the hypothesis?

A. Phalaropes rarely feed while on land.

B. A given phalarope spins exclusively either to the right or to the left.

C. Phalaropes sometimes spin when they are not feeding.

D. Different phalaropes spin at somewhat different rates.

E. Phalaropes do not usually spin when food is abundant at the surface.

56. A city plans to attract new citizens with new housing and new facilities such as parks, recreation centers and

libraries. One component of the city's plans is to require that development seeking permission is to build this new

housing to provide these additional facilities at no cost to the city.

Which of the following, if true, would point to a possible flaw in the city's plan?

(A) Developers would pass along their costs to the buyer; thereby raising the cost of housing units beyond the

ability of likely purchasers to afford them.

(B) Light, nonpolluting industries have located in the area, offering more jobs and better-paying jobs than do the

more established industries in the area.

Page 22: Critical Reasoning Questions - CATKing Educare · Critical Reasoning Questions . Borivali | Andheri |Powai & Pune Contact : 9029077114/ 8097607504 / 9619160261 / 8149774534 1. Homeowners

Borivali | Andheri |Powai & Pune

Contact : 9029077114/ 8097607504 / 9619160261 / 8149774534

www.catking.in

(C) Other towns and cities nearby have yet to embark on comparable plans to attract new citizens

(D) Most developers see the extra expense of providing municipal facilities as simply one of the many costs of

doing business.

(E) Studies show that purchasers of new houses, especially first-time buyers, rank recreational resources as an

important factor in deciding to buy a particular house.

57. In order to reduce the number of items damaged while in transit to customers, packaging consultants

recommended that the TrueSave mail-order company increase the amount of packing material so as to fill any

empty spaces in its cartons. Accordingly, TrueSave officials instructed the company’s packers to use more packing

material than before, and the packers zealously acted on these instructions and used as much as they could.

Nevertheless, customer reports of damaged items rose somewhat.

Which of the following, if true, most helps to explain why acting on the consultants’ recommendation failed to

achieve its goal?

A. The change in packing policy led to an increase in expenditure on packing material and labor.

B. When packing material is compressed too densely, it loses some of its capacity to absorb shock.

C. The amount of packing material used in a carton does not significantly influence the ease with which a

customer can unpack the package.

D. Most of the goods that TrueSave ships are electronic products that are highly vulnerable to being damaged in

transit.

E. TrueSave has lost some of its regular customers as a result of the high number of damaged items they received.

58. It’s time we stopped searching for new statistics to suggest that we are not spending enough on education. In fact,

education spending increased 30 percent overall during the last decade.

Which of the following, if true, would most weaken the argument above?

(A) Despite increased spending on education, enrollment in our elementary and secondary schools declined about

4 percent during the last ten years.

(B) Our spending on gasoline increased more than 100 percent during the last decade.

(C) When adjusted for inflation, our per-pupil expenditure on education this year is less than it was ten years ago.

(D) Eleven other economically developed nations spend more on education than we do.

(E) The achievement levels of our students have been declining steadily since 1960, and the last decade produced

no reversal in this trend.

59. Albinism is a rare genetic condition that inhibits the production of melanin, or pigmentation, in the skin and hair.

People born with albinism are unusually susceptible to sunburn, melanoma, and a range of other health issues

that are generally connected to excessive exposure to the sun.

The statements above, if true, provide the most support for which of the following conclusions?

Page 23: Critical Reasoning Questions - CATKing Educare · Critical Reasoning Questions . Borivali | Andheri |Powai & Pune Contact : 9029077114/ 8097607504 / 9619160261 / 8149774534 1. Homeowners

Borivali | Andheri |Powai & Pune

Contact : 9029077114/ 8097607504 / 9619160261 / 8149774534

www.catking.in

A)People born with albinism develop other biological protections against melanoma and other sun-related health

issues.

B)Humans with a high production of melanin can easily ignore health issues related to exposure to the sun.

C)When a non-albino person gets sunburn, the amount of melanin produced by that person decreases.

D)In humans, melanin plays a role in protecting the skin from developing sunburn and other sun-related ailments.

E)It is not possible for a person born with albinism to adopt other artificial protective measures against excessive

exposure to the sun.

60. Ferber's syndrome, a viral disease that frequently affects cattle, is transmitted to these animals through infected

feed. Even though chickens commercially raised for meat are often fed the type of feed identified as the source of

infection in cattle, Ferber's syndrome is only rarely observed in chickens. This fact, however, does not indicate

that most chickens are immune to the virus that causes Ferber's syndrome, since _____________.

Which of the following most logically completes the argument?

(A) chickens and cattle are not the only kinds of farm animal that are typically fed the type of feed liable to be

contaminated with the virus that causes Ferber's syndrome

(B) Ferber's syndrome has been found in animals that have not been fed the type of feed liable to be contaminated

with the virus that can cause the disease

(C) resistance to some infectious organisms such as the virus that causes Ferber's syndrome can be acquired by

exposure to a closely related infectious organism

(D) chickens and cattle take more than a year to show symptoms of Ferber's syndrome, and chickens

commercially raised for meat, unlike cattle, are generally brought to market during the first year of life

(E) the type of feed liable to be infected with the virus that causes Ferber's syndrome generally constitutes a

larger proportion of the diet of commercially raised chickens than of commercially raised cattle

61. While many people think of genetic manipulation of food crops as being aimed at developing larger and larger

plant varieties, some plant breeders have in fact concentrated on discovering or producing dwarf varieties, which

are roughly half as tall as normal varieties.

Which of the following would, if true, most help to explain the strategy of the plant breeders referred to above?

(A) Plant varieties used as food by some are used as ornamentals by others.

(B) The wholesale prices of a given crop decrease as the supply of it increases.

(C) Crops once produced exclusively for human consumption are often now used for animal feed.

(D) Short plants are less vulnerable to strong wind and heavy rains.

(E) Nations with large industrial sectors tend to consume more processed grains.

62. Trancorp currently transports all its goods to Burland Island by truck. The only bridge over the channel

separating Burland from the mainland is congested, and trucks typically spend hours in traffic. Trains can reach

the channel more quickly than trucks, and freight cars can be transported to Burland by barges that typically cross

the channel in an hour. Therefore, to reduce shipping time, Trancorp plans to switch to trains and barges to

transport goods to Burland.

Page 24: Critical Reasoning Questions - CATKing Educare · Critical Reasoning Questions . Borivali | Andheri |Powai & Pune Contact : 9029077114/ 8097607504 / 9619160261 / 8149774534 1. Homeowners

Borivali | Andheri |Powai & Pune

Contact : 9029077114/ 8097607504 / 9619160261 / 8149774534

www.catking.in

Which of the following would be most important to know in determining whether Trancorp’s plan, if

implemented, is likely to achieve its goal?

A: Whether transportation by train and barge would be substantially less expensive than transportation by truck.

B: Whether there are boats that can make the trip between the mainland and Burland faster than barges can

C: Whether loading the freight cars onto barges is very time consuming.

D: Whether the average number of vehicles traveling over the bridge into Burland has been relatively constant in

recent years?

E: Whether most trucks transporting goods into Burland return to the mainland empty

63. Scientists propose placing seismic stations on the floor of the Pacific Ocean to warn threatened coastal

communities on the northwestern coast of the United States of approaching tidal waves caused by earthquakes.

Since forewarned communities could take steps to evacuate, many of the injuries and deaths that would otherwise

occur could be avoided if the government would implement this proposal.

The answer to which of the following questions would be most important in determining whether implementing

the proposal would be likely to achieve the desired result?

(A) When was the last time that the coastal communities were threatened by an approaching tidal wave?

(B) How far below sea level would the stations be located?

(C) Would there be enough time after receiving warning of an approaching tidal wave for communities to

evacuate safely?

(D) How soon after a tidal wave hits land is it safe for evacuees to return to their communities?

(E) Can the stations be equipped to collect and relay information about phenomena other than tidal waves caused

by earthquakes?

64. Certain messenger molecules fight damage to the lungs from noxious air by telling the muscle cells encircling the

lungs’ airways to contract. This partially seals off the lungs. An asthma attack occurs when the messenger

molecules are activated unnecessarily, in response to harmless things like pollen or household dust.

Which of the following, if true, points to the most serious flaw of a plan to develop a medication that would

prevent asthma attacks by blocking receipt of any messages sent by the messenger molecules referred to above?

(A) Researchers do not yet know how the body produces the messenger molecules that trigger asthma attacks.

(B) Researchers do not yet know what makes one person’s messenger molecules more easily activated than

another’s.

(C) Such a medication would not become available for several years, because of long lead times in both

development and manufacture.

(D) Such a medication would be unable to distinguish between messages triggered by pollen and household dust

and messages triggered by noxious air.

(E) Such a medication would be a preventative only and would be unable to alleviate an asthma attack once it had

started

Page 25: Critical Reasoning Questions - CATKing Educare · Critical Reasoning Questions . Borivali | Andheri |Powai & Pune Contact : 9029077114/ 8097607504 / 9619160261 / 8149774534 1. Homeowners

Borivali | Andheri |Powai & Pune

Contact : 9029077114/ 8097607504 / 9619160261 / 8149774534

www.catking.in

65. A report on acid rain concluded, “Most forests in Canada are not being damaged by acid rain.” Critics of the report

insist the conclusion be changed to, “Most forests in Canada do not show visible symptoms of damage by acid rain,

such as abnormal loss of leaves, slower rates of growth, or higher mortality.”

Which of the following, if true, provides the best logical justification for the critics’ insistence that the

report’s conclusion be changed?

(A) Some forests in Canada are being damaged by acid rain.

(B) Acid rain could be causing damage for which symptoms have not yet become visible.

(C) The report does not compare acid rain damage to Canadian forests with acid rain damage to forests in other

countries.

(D) All forests in Canada have received acid rain during the past fifteen years.

(E) The severity of damage by acid rain differs from forest to forest.

66. A report on acid rain concluded, “Most forests in Canada are not being damaged by acid rain.” Critics of the report

insist the conclusion be changed to, “Most forests in Canada do not show visible symptoms of damage by acid rain,

such as abnormal loss of leaves, slower rates of growth, or higher mortality.”

Which of the following, if true, provides the best logical justification for the critics’ insistence that the

report’s conclusion be changed?

(A) Some forests in Canada are being damaged by acid rain.

(B) Acid rain could be causing damage for which symptoms have not yet become visible.

(C) The report does not compare acid rain damage to Canadian forests with acid rain damage to forests in other

countries.

(D) All forests in Canada have received acid rain during the past fifteen years.

(E) The severity of damage by acid rain differs from forest to forest.

67. The sustained massive use of pesticides in farming has two effects that are especially pernicious. First, it often

kills off the pests' natural enemies in the area . Second, it often unintentionally gives rise to insecticide-resistant

pests, since those insects that survive a particular insecticide will be the ones most resistant to it, and they are the

ones left to breed.

From the passage above, it can be properly inferred that the effectiveness of the sustained massive use of

pesticides can be extended by doing which of the following, assuming that each is a realistic possibility?

(Al Using only chemically stable insecticides

(B) Periodically switching the type of insecticide used

(C) Gradually increasing the quantities of pesticides used

(D) Leaving a few fields fallow every year

(E) Breeding higher-yielding varieties of crop plants

Page 26: Critical Reasoning Questions - CATKing Educare · Critical Reasoning Questions . Borivali | Andheri |Powai & Pune Contact : 9029077114/ 8097607504 / 9619160261 / 8149774534 1. Homeowners

Borivali | Andheri |Powai & Pune

Contact : 9029077114/ 8097607504 / 9619160261 / 8149774534

www.catking.in

68. Although the number of large artificial satellites orbiting the Earth is small compared to the number of small

pieces of debris in orbit, the large satellites interfere more seriously with telescope observations because of the

strong reflections they produce. Because many of those large satellites have ceased to function, the proposal has

recently been made to eliminate interference from nonfunctioning satellites by exploding them in space.

This proposal, however, is ill conceived, since _______.

A. many nonfunctioning satellites remain in orbit for years

B. for satellites that have ceased to function, repairing them while they are in orbit would be prohibitively

expensive

C. there are no known previous instances of satellites’ having been exploded on purpose

D. the only way to make telescope observations without any interference from debris in orbit is to use telescopes

launched into extremely high orbits around the Earth

E. a greatly increased number of small particles in Earth’s orbit would result in a blanket of reflections that would

make certain valuable telescope observations impossible

69. A milepost on the towpath read “21” on the side facing the hiker as she approached it and “23” on its back.

She reasoned that the next milepost forward on the path would indicate that she was halfway between

one end of the path and the other. However, the milepost one mile further on read “20” facing her and “24”

behind.

Which of the following, if true, would explain the discrepancy described above?

(A) The numbers on the next milepost had been reversed.

(B) The numbers on the mileposts indicate kilometers, not miles.

(C) The facing numbers indicate miles to the end of the path, not miles from the beginning.

(D) A milepost was missing between the two the hiker encountered.

(E) The mileposts had originally been put in place for the use of mountain bikers, not for hikers.

70. “ on the whole,” Ms. Dennis remarked, “ engineering students are lazier now than they used to be. I know because

fewer and fewer of my students regularly do the work they are assigned.

The conclusion drawn above depends on which of the following assumptions?

a. Engineering students are working less because, in a booming market, they are spending more and more time

investigating different job opportunities.

b. Whether or not students do the work they are assigned is a good indication of how lazy they are.

c. Engineering students should work harder than students in less demanding fields.

d. Ms. Dennis’ students are doing less work because Ms. Dennis is not as effective a teacher as she once was.

e. Laziness is something most people do not outgrow.

71. In recent years many cabinetmakers have been winning acclaim as artists. But since furniture must be useful,

cabinetmakers must exercise their craft with an eye to the practical utility of their product. For this reason,

cabinetmaking is not art.

Page 27: Critical Reasoning Questions - CATKing Educare · Critical Reasoning Questions . Borivali | Andheri |Powai & Pune Contact : 9029077114/ 8097607504 / 9619160261 / 8149774534 1. Homeowners

Borivali | Andheri |Powai & Pune

Contact : 9029077114/ 8097607504 / 9619160261 / 8149774534

www.catking.in

Which of the following is an assumption that supports drawing the conclusion above from the reason given for

that conclusion?

A. Some furniture is made to be placed in museums, where it will not be used by anyone.

B. Some cabinetmakers are more concerned than others with the practical utility of the products they produce.

C. Cabinetmakers should be more concerned with the practical utility of their products than they currently are.

D. An object is not an art object if its maker pays attention to the objects practical utility.

E. Artists are not concerned with the monetary value of their products.

72. Albinism is a rare genetic condition that inhibits the production of melanin, or pigmentation, in the skin and hair.

People born with albinism are unusually susceptible to sunburn, melanoma, and a range of other health issues

that are generally connected to excessive exposure to the sun. The statements above, if true, provide the most

support for which of the following conclusions?

A. People born with albinism develop other biological protections against melanoma and other sun-related health

issues.

B. Humans with a high production of melanin can easily ignore health issues related to exposure to the sun.

C. When a non-albino person gets sunburn, the amount of melanin produced by that person decreases.

D. In humans, melanin plays a role in protecting the skin from developing sunburn and other sun-related ailments.

E. It is not possible for a person born with albinism to adopt other artificial protective measures against excessive

exposure to the sun.

73. The climbing season of 2006 was the deadliest on record for those attempting to conquer Mount Everest, the

world’s tallest mountain, as more people perished attempting to reach the summit in 2006 than in any other year.

Interestingly, most mountaineering experts attribute the high number of fatalities, almost all of which occurred in

the unforgiving “death-zone” above 26,000 feet, directly to the exceptionally good weather prevalent during the

2006 climbing season. Which of the following, if true, best helps to explain the conclusion of the mountaineering

experts?

A. All of the forecasts were for extremely bad weather; the good weather was a significant surprise to all of the

climbers.

B. The good weather prompted significantly more people than ever to try to reach the summit and enter the

“deathzone,” many of whom would have turned back at a lower altitude in poorer weather.

C. The good weather caused the “death-zone” to have warmer temperatures and less intense winds than in recent

years.

D. Modern equipment is particularly effective in protecting climbers from the elements in bad weather.

E. Many accomplished climbers don’t attempt Mt. Everest during good weather because they feel it is not a

challenge.

Page 28: Critical Reasoning Questions - CATKing Educare · Critical Reasoning Questions . Borivali | Andheri |Powai & Pune Contact : 9029077114/ 8097607504 / 9619160261 / 8149774534 1. Homeowners

Borivali | Andheri |Powai & Pune

Contact : 9029077114/ 8097607504 / 9619160261 / 8149774534

www.catking.in

74. At any given time, approximately fifteen percent of all homes in Florida are on the market. In Texas, however, only

seven percent of all homes are on the market at any given time. Therefore, one will have a wider selection of

homes to choose from if one looks for a home in Florida rather than in Texas. Which of the following, if true, would

most seriously strengthen the argument above?

A) Homes in Florida tend to be less expensive than those in Texas.

B) Mortgages are easier to obtain for homes in Florida than for homes in Texas.

C) The construction industry in Texas has reported significant growth over the past year.

D) The cost of constructing new homes in Texas is higher than in Florida.

E) The total number of homes in Florida is three times greater than the total number in Texas.

75. Advertisement: Last year, Factorial Mutual Fund continued its strong record of investment performance.

Investors who included Factorial Fund in their portfolios realized an average capital gain of 15% across all of their

investment assets, nearly double the market return. Therefore, investors interested in high returns should

consider adding Factorial Mutual Fund to their portfolios.

The claim in the advertisement above is based on which of the following assumptions?

A) Factorial Fund is one of the oldest and largest mutual funds in the investment industry.

B) A substantial part of the 15% portfolio appreciation was attributable to the returns of Factorial Fund and this

level of fund’s performance is likely to persist in the future.

C) Last year, Factorial Fund outperformed all other funds with similar style and investment objectives.

D) The Fund is suitable to all investors.

E) The fees charged by Factorial Fund are among the lowest in the investment industry.

76. A certain mobile smartphone manufacturer aims to increase its market share by deeply discounting its

smartphone prices for the next several months. The discounts will cut into profits, but because they will be heavily

advertised the manufacturer hopes that they will attract buyers away from rival manufacturers' smartphones. In

the longer term, the smartphone manufacturer envisions that customers initially attracted by the discounts may

become loyal customers.

In assessing the plan's chances of achieving its aim, it would be most useful to know which of the following?

(A) Whether the smartphone's competitors are likely to respond by offering deep discounts on their own products

(B) Whether the advertisements will be created by the manufacturer's current advertising agency

(C) Whether some of the smartphone manufacturers' models will be more deeply discounted than others

(D) Whether the smartphone manufacturer will be able to cut costs sufficiently to maintain profit margins even

when the discounts are in effect

(E) Whether an alternative strategy might enable the smartphone manufacturer to enhance its profitability while

holding a constant or diminishing share of the market

Page 29: Critical Reasoning Questions - CATKing Educare · Critical Reasoning Questions . Borivali | Andheri |Powai & Pune Contact : 9029077114/ 8097607504 / 9619160261 / 8149774534 1. Homeowners

Borivali | Andheri |Powai & Pune

Contact : 9029077114/ 8097607504 / 9619160261 / 8149774534

www.catking.in

77. By competing with rodents for seeds, black ants help control rodent populations that pose a public health risk.

However, a very aggressive species of blank ant, the Loma ant, which has recently invaded a certain region, has a

venomous sting that is often fatal to humans. Therefore, the planned introduction into that region of ant flies,

which prey on Loma ants, would benefit public health.

Which of the following, if true, most strengthens the argument?

A. Ant flies do not attack black ants other than Loma ants.

B. Loma ants are less effective than many bird species in competing with rodents for seeds.

C. Certain other species of black ants are more effective than Loma ants in competing with rodents for seeds.

D. The sting of Loma ants can also be fatal to rodents.

E. The use of pesticides to control Loma ants could have harmful effects on the environment.

78. Company Alpha buys free-travel coupons from people who are awarded the coupons by Bravo Airlines for flying

frequently on Bravo airplanes. The coupons are sold to people who pay less for the coupons than they would pay

by purchasing tickets from Bravo. This marketing of coupons results in lost revenue for Bravo.

To discourage the buying and selling of free-travel coupons, it would be best for Bravo Airlines to restrict

the

(A) number of coupons that a person can be awarded in a particular year

(B) use of the coupons to those who were awarded the coupons and members of their immediate families

(C) days that the coupons can be used to Monday through Friday

(D) amount of time that the coupons can be used after they are issued

(E) number of routes on which travelers can use the coupons

79. After its customers complained about being pressured to buy unneeded insurance, an insurance

agency stopped rewarding its agents for high sales volume and instead gave them bonuses for high

levels of customer satisfaction. Under this new plan, both customer satisfaction and the insurance

agency's sales increased.

Each of the following, if true, helps to explain how the change in incentives for agents could have

resulted in increased sales EXCEPT:

A) Customers were so pleased that the insurance agency had responded to their complaints that they

recommended the agency to their friends.

B) Agents listened more closely to customers of long standing and were able to sell them additional insurance

policies that met new needs.

C) Agents more frequently postponed completing the attendant paperwork even after the terms for an insurance

policy were settled to the satisfaction of the client.

D) Dissatisfied customers of other agencies, attracted by the reports of the change in agency policy, became

customers of the agency.

Page 30: Critical Reasoning Questions - CATKing Educare · Critical Reasoning Questions . Borivali | Andheri |Powai & Pune Contact : 9029077114/ 8097607504 / 9619160261 / 8149774534 1. Homeowners

Borivali | Andheri |Powai & Pune

Contact : 9029077114/ 8097607504 / 9619160261 / 8149774534

www.catking.in

E) Having come to trust the increased judiciousness of the agents' recommendations, customers approached the

agency to discuss and ultimately to buy more supplementary insurance than they previously had bought under

pressure.

80. Surveys consistently show that the best-selling ice cream flavor is vanilla, although those

who prefer chocolate rarely order vanilla. Vanilla-flavored candy, then, probably sells better

than chocolate-flavored candy. Which of the following is an assumption upon which the

author of the argument relies?

A) Because someone prefers vanilla ice cream does not mean he prefers vanilla-flavored candy.

B) Children who prefer vanilla ice cream also tend to like chocolate candy.

C) Those who prefer neither vanilla nor chocolate ice cream also prefer other flavors of candy.

D) Someone who prefers vanilla ice cream may still order chocolate on occasion.

E) Preferences for certain ice cream flavors are similar to preferences for candy flavors.

81. Many states seek to reduce their budget deficits by cutting state funding for colleges and universities. However,

many students who cannot afford the tuition increases resulting from the decreased state funding will not be able

to earn a college education and, as a result, will not be able to move into higher paying careers. Consequently, the

state will lose future income tax revenues.

Which of the following, if true, most strengthens the argument above?

A. Many states that have already reduced their funding for colleges and universities have seen declines in income

tax revenues in the following years.

B. Many states that have not cut education funding have seen budget deficits grow larger.

C. States that do not cut their education funding sometimes also experience declines in tax revenues

D. Cuts to colleges and universities affect two-year colleges and well as four-year colleges.

E. The size of the budget deficit is usually too great to be eliminated by reductions in higher education funding

alone.

82. Many students enter the business school application process with excellent undergraduate grades. Since excellent

undergraduate grades require passing frequent tests and final examinations, these students must have learned

very strong test-taking techniques either before or during their undergraduate educational careers. Despite this,

many students with promising grades fail to get scores on the GMAT that match the level of performance

displayed by their undergraduate careers.

The apparent discontinuity between undergraduate academic performance and GMAT test results could best be

explained by which of the following suppositions?

A) Many students with strong undergraduate careers aspire to get accepted into a top tier business school and,

therefore, feel that they must present not only solid grades but also a flawless GMAT score.

B) The GMAT tests quantitative and verbal skills, both of which are covered to some extent in an undergraduate

Page 31: Critical Reasoning Questions - CATKing Educare · Critical Reasoning Questions . Borivali | Andheri |Powai & Pune Contact : 9029077114/ 8097607504 / 9619160261 / 8149774534 1. Homeowners

Borivali | Andheri |Powai & Pune

Contact : 9029077114/ 8097607504 / 9619160261 / 8149774534

www.catking.in

curriculum.

C) The skills necessary to excel on most academically derived exams are often learned in study-skills workshops

held by colleges, from peer tutors, or from written feedback provided by professors and teaching assistants,

whereas the skills required for the GMAT are often learned in classes or from books dedicated to preparing for the

exam.

D) Over 75% of business school applicants report that they found it necessary to study for the GMAT, using

computer software, books, and/or specialty classes.

E) The skills and strategies required for excellence on the GMAT and other standardized tests are often quite

divergent from those required for success on traditional academic exams.

83. An overly centralized economy, not the changes in the climate, is responsible for the poor agricultural production

in country x since its new government came to power. Neighboring country y has experienced the same climate

conditions, but while agricultural production has been falling in country x, it has been rising in country.

Which of the following, if true, would most weaken the argument?

(a) Industrial production also is declining in country x.

(b) Whereas country y is landlocked, country x has a major seaport.

(c) Both country x and country y have been experiencing drought conditions.

(d) The crops that have always been grown in country x are different from those that have always been grown in

country y.

(e) Country x's new government instituted a centralized economy with the intention of ensuring an equitable

distribution of goods.

84. The local university recently hired a new soccer coach. Although she has several years' worth of coaching

experience and is a diligent student of the game, she was never a member of a collegiate soccer team. For this

reason, the new coach will be unable to build a successful program.

The argument above is based on which of the following assumptions?

(A) The local university should have hired a former collegiate soccer player as its new coach.

(B) Coaching experience is one of the most crucial factors for coaching success.

(C) The previous coach at the university was dismissed due to her lack of success,

(D) To build a successful soccer program as a coach, one must be a former collegiate soccer player.

(E) The university does not plan to provide the new coach with the resources necessary to build a successful

program.

85. In an attempt to promote the widespread use of paper rather than plastic, and thus reduce non biodegradable

waste, the council of a small town plans to ban the sale of disposable plastic goods for which substitutes made of

paper exist. The council argues that since most paper is entirely biodegradable, paper goods are environmentally

preferable.

Page 32: Critical Reasoning Questions - CATKing Educare · Critical Reasoning Questions . Borivali | Andheri |Powai & Pune Contact : 9029077114/ 8097607504 / 9619160261 / 8149774534 1. Homeowners

Borivali | Andheri |Powai & Pune

Contact : 9029077114/ 8097607504 / 9619160261 / 8149774534

www.catking.in

Which of the following, if true, indicates that the plan to ban the sale of disposable plastic goods is ill suited to the

town council’s environmental goals?

(A) Although biodegradable plastic goods are now available, members of the town council believe biodegradable

paper goods to be safer for the environment.

(B) The paper factory at which most of the townspeople are employed plans to increase production of

biodegradable paper goods.

(C) After other towns enacted similar bans on the sale of plastic goods, the environmental benefits were not

discernible for several years.

(D) Since most townspeople prefer plastic goods to paper goods in many instances, they are likely to purchase

them in neighboring towns where plastic goods are available for sale.

(E) Products other than those derived from wood pulp are often used in the manufacture of paper goods that are

entirely biodegradable.

86. Traverton’s city council wants to minimize the city’s average yearly expenditures on its traffic signal lights and so

is considering replacing the incandescent bulbs currently in use with arrays of light-emitting diodes (LEDs) as the

incandescent bulbs burn out. Compared to incandescent bulbs, LED arrays consume significantly less energy and

cost no more to purchase. Moreover, the costs associated with the conversion of existing fixtures so as to accept

LED arrays would be minimal.

Which of the following would it be most useful to know in determining whether switching to LED arrays would be

likely to help minimize Traverton’s yearly maintenance costs?

(A) Whether the expected service life of LED arrays is at least as long as that of the currently used incandescent

bulbs

(B) Whether any cities have switched from incandescent lights in their traffic signals to lighting elements other

than LED arrays

(C) Whether the company from which Traverton currently buys incandescent bulbs for traffic signals also sells

LED arrays

(D) Whether Traverton’s city council plans to increase the number of traffic signal lights in Traverton

(E) Whether the crews that currently replace incandescent bulbs in Traverton’s traffic signals know how to

convert the existing fixtures so as to accept LED arrays

87. Bevex, an artificial sweetener used only in soft drinks, is carcinogenic for mice, but only when it is consumed in

very large quantities. To ingest an amount of Bevex equivalent to the amount fed to the mice in the relevant

studies, a person would have to drink 25 cans of Bevex-sweetened soft drinks per day. For that reason, Bevex is in

fact safe for people.

For which of the following assumptions does the argument that Bevex is safe for people depend?

Page 33: Critical Reasoning Questions - CATKing Educare · Critical Reasoning Questions . Borivali | Andheri |Powai & Pune Contact : 9029077114/ 8097607504 / 9619160261 / 8149774534 1. Homeowners

Borivali | Andheri |Powai & Pune

Contact : 9029077114/ 8097607504 / 9619160261 / 8149774534

www.catking.in

(A)Cancer from carcinogenic substances develops more slowly in mice than it does in people.

(B)If all food additives that are currently used in foods were tested, some would be found to be carcinogenic for

mice.

(C)People drink fewer than 25 cans of Bevex-sweetened soda per day.

(D)People can obtain important health benefits by controlling their weight through the use of artificially

sweetened soft drinks.

(E)Some of the studies done on Bevex were not relevant to the question of whether or not Bevex is carcinogenic

for people.

88. Only a reduction of 10 percent in the number of scheduled flights using Greentown's airport will allow the delays

that are so common there to be avoided. Hevelia airstrip, 40 miles away, would, if upgraded and expanded, be an

attractive alternative for fully 20 percent of the passengers using Greentown airport. Nevertheless, experts reject

the claim that turning Hevelia into a full-service airport would end the chronic delays at Greentown.

Which of the following, if true, most helps to justify the experts' position?

(A) Turning Hevelia into a full-service airport would require not only substantial construction at the airport itself,

but also the construction of new access highways.

(B) A second largely undeveloped airstrip close to Greentown airport would be a more attractive alternative than

Hevelia for many passengers who now use Greentown.

(C) Hevelia airstrip lies in a relatively undeveloped area but would, if it became a full-service airport, be a magnet

for commerc ial and residential development.

(D) If an airplane has to wait to land, the extra jet fuel required adds significantly to the airline's costs.

(E) Several airlines use Greentown as a regional hub, so that most flights landing at Greentown have many

passengers who then take different flights to reach their final destinations.

89. The country of Ertland has never imported apples in any significant quantity because consumers there generally

prefer the unique texture of Ertland-grown apples. Nevertheless, apple growers from Kosolia, a neighboring

country, plan to sell their apples in Ertland by selling Kosolia-grown apples at half the price of local apples and

promoting them as a nourishing, low-cost alternative.

Which of the following, if true, casts most doubt on the viability of the plan by Kosolia's apple growers to sell their

apples in Ertland?

A) Most of the varieties of apples grown in Ertland were originally derived from common Kosolian varieties.

B) Consumers in Ertland tend to spend about the same proportion of their income on fresh fruits and vegetables

as do consumers in Kosolia.

C) At times in the past, Ertland has exported significant quantities of apples to Kosolia.

D) Some varieties of apples grown in Kosolia can be harvested throughout most of the year, whereas the varieties

grown in Ertland can be harvested only during two months of the year.

Page 34: Critical Reasoning Questions - CATKing Educare · Critical Reasoning Questions . Borivali | Andheri |Powai & Pune Contact : 9029077114/ 8097607504 / 9619160261 / 8149774534 1. Homeowners

Borivali | Andheri |Powai & Pune

Contact : 9029077114/ 8097607504 / 9619160261 / 8149774534

www.catking.in

E) Profit of Ertland-grown apples are high enough in Ertland that growers, wholesalers, and retailers there could

easily afford to reduce the price at which these apples are sold.

90. When interest rates are high, insurance companies reduce the premiums they charge for many kinds of insurance

policies. The reason is that insurance companies want to take in as much money as possible in premiums so that

they can invest the money at high rates of interest. And premium reductions help achieve this objective, since

__________.

Which of the following most logically completes the argument below?

(A) interest rates are likely to decrease when large amounts of money are available for loans

(B) smaller insurance companies are not able to amass enough money to take advantage of investing at high

interest rates

(C) insurance companies can sell many more insurance policies if they charge lower premiums than they would if

they left premiums unchanged

(D) an increase in the number of policies sold eventually leads to an increase in the number of claims that an

insurance company has to pay

(E) the number of claims that insurance companies pay increases at a higher rate than does the number of policies

that the insurance companies can sell at the lower premiums

91. Astronomer: Most stars are born in groups of thousands, each star in a group forming from the same parent cloud

of gas. Each cloud has a unique homogeneous chemical composition. Therefore whenever two stars have the same

chemical composition as each other, they must have originated from the same cloud of gas. ( C )

Which of the following, if true, would most strengthen the astronomer's argument?

(A) In some groups of stars, not every star originated from the same parent cloud of gas.

(B) Clouds of gas of similar or identical chemical composition may be remote from each other.

(C) Whenever a star forms, it inherits the chemical composition of its parent cloud of gas.

(D) Many stars in vastly different parts of the universe are quite similar in their chemical compositions.

(E) Astronomers can at least sometimes precisely determine whether a star has the same chemical composition as

its parent cloud of gas.

92. For similar cars and comparable drivers, automobile insurance for collision damage has always cost more

in Greatport than in Fairmont. Police studies, however, show that cars owned by Greatport residents are, on

average, slightly less likely to be involved in a collision than cars in Fairmont. Clearly, therefore, insurance

companies are making a greater profit on collisiondamage insurance in Greatport than in Fairmont.

In evaluating the argument, it would be most useful to compare

Page 35: Critical Reasoning Questions - CATKing Educare · Critical Reasoning Questions . Borivali | Andheri |Powai & Pune Contact : 9029077114/ 8097607504 / 9619160261 / 8149774534 1. Homeowners

Borivali | Andheri |Powai & Pune

Contact : 9029077114/ 8097607504 / 9619160261 / 8149774534

www.catking.in

(A) the level of traffic congestion in Greatport with the level of traffic congestion in Fairmont

(B) the cost of repairing collision damage in Greatport with the cost of repairing collision damage in Fairmont

(C) the rates Greatport residents pay for other forms of insurance with the rates paid for similar insurance by

residents of Fairmont

(D) the condition of Greatport's roads and streets with the condition of Fairmont's roads and streets

(E) the cost of collision-damage insurance in Greatport and Fairmont with that in other cities

93. For the past several years, a certain technology has been widely used to transmit data among networked

computers. Recently two data transmission companies, Aptron and Gammatech, have each developed separate

systems that allow network data transmission at rates ten times faster than the current technology allows.

Although the systems are similarly priced and are equally easy to use, Aptron's product is likely to dominate the

market, because __________.

Which of the following most logically completes the passage?

(A) Gammatech has been in the business of designing data transmission systems for several years more than

Aptron has

(B) the number of small businesses that need computer networking systems is likely to double over the next few

years

(C) it is much more likely that Gammatech's system will be expandable to meet future needs

(D) unlike many data transmission companies, Aptron and Gammatech develop computers in addition to data

transmission systems

(E) it is easier for users of the current data transmission technology to switch to Aptron's product than to

Gammatech's

94. The Smithtown Theatre, which stages old plays, has announced an expansion that will double its capacity along

with its operating costs. The theatre is only slightly profitable at present. In addition, all of the current customers

live in Smithtown, and the population of the town is not expected to increase in the next several years. Thus, the

expansion of the Smithtown Theatre will prove unprofitable.

Which of the following, if true, would most seriously weaken the argument?

(A) A large movie chain plans to open a new multiplex location in Smithtown later this year.

(B) Concession sales in the Smithtown Theatre comprise a substantial proportion of the theatre's revenues.

(C) Many recent arrivals to Smithtown are students that are less likely to attend the Smithtown Theatre than are

older residents.

(D) The expansion would allow the Smithtown Theatre to stage larger, more popular shows that will attract

customers from neighboring towns.

(E) The Board of the Smithtown Theatre often solicits input from residents of the town when choosing which

shows to stage.

Page 36: Critical Reasoning Questions - CATKing Educare · Critical Reasoning Questions . Borivali | Andheri |Powai & Pune Contact : 9029077114/ 8097607504 / 9619160261 / 8149774534 1. Homeowners

Borivali | Andheri |Powai & Pune

Contact : 9029077114/ 8097607504 / 9619160261 / 8149774534

www.catking.in

95. To evaluate a plan to save money on office space expenditures by having its employees work at home, XYZ

Company asked volunteers from its staff to try the assignment for six months. During this period, the productivity

of these employees was as high or higher than before.

Which of the following, if true, would argue most strongly against deciding on the basis of the trial results,

to implement the company's plan?

(A) The employees who agreed to participate in the test of the plan were among the company's most self-

motivated and independent workers.

(B) The savings that would accrue from reduced office-space expenditures alone would be sufficient to justify the

arrangement for the company.

(C) Other companies that have achieved successful results from work-at-home plans have work forces that are

substantially lower than that of XYZ.

(D) The volunteers who worked at home were able to communicate with other employees as necessary for

performing at work.

(E) Minor changes in the way office work is organized at XYZ would yield increases in employee productivity

similar to those achieved in the trial

96. A computer equipped with signature-recognition software, which restricts access to a computer to those people

whose signatures are on file, identifies a person’s signature by analyzing not only the form of the signature but

also such characteristics as pen pressure and signing speed. Even the most adept forgers cannot duplicate all of

the characteristics the program analyzes.

Which of the following can be logically concluded from the passage above?

(A) The time it takes to record and analyze a signature makes the software impractical for everyday use.

(B) Computers equipped with the software will soon be installed in most banks.

(C) Nobody can gain access to a computer equipped with the software solely by virtue of skill at forging

signatures.

(D) Signature-recognition software has taken many years to develop and perfect.

(E) In many cases even authorized users are denied legitimate access to computers equipped with the software.

97. Most young employees move quickly from one company to the next, changing jobs up to three times in a year.

However, Happy Snax is known for retaining young workers for well over a year. Happy Snax claims its retention

rate is due to the fact that they do not require uniforms or formal break times.

Which of the following, if true, most strengthens Happy Snax explanation of its success in retaining young

employees?

A) Young employees often change jobs because they become bored with their current job and are looking for

Page 37: Critical Reasoning Questions - CATKing Educare · Critical Reasoning Questions . Borivali | Andheri |Powai & Pune Contact : 9029077114/ 8097607504 / 9619160261 / 8149774534 1. Homeowners

Borivali | Andheri |Powai & Pune

Contact : 9029077114/ 8097607504 / 9619160261 / 8149774534

www.catking.in

something new and exciting

B) Many young employees at Happy Snax have not had a job previously

C) Young employees find uniforms and formal schedules unappealing

D) Uniforms and formal schedules lead to more productivity in the workplace

E) Happy Snax also provides free housing to all of its young employees for the first year

98. Although custom prosthetic bone replacements produced through a new computer-aided design process will cost

more than twice as much as ordinary replacements, custom replacements should still be cost-effective. Not only

will surgery and recovery time be reduced, but custom replacements should last longer; thereby reducing the

need for further hospital stays.

Which of the following must be studied in order to evaluate the argument presented above?

(A) The amount of time a patient spends in surgery versus the amount of time spent recovering from surgery

(B) The amount by which the cost of producing custom replacements has declined with the introduction of the

new technique for producing them

(C) The degree to which the use of custom replacements is likely to reduce the need for repeat surgery when

compared with the use of ordinary replacements

(D) The degree to which custom replacements produced with the new technique are more carefully manufactured

than are ordinary replacements

(E) The amount by which custom replacements produced with the new technique will drop in cost as the

production procedures become standardized and applicable on a larger scale

99. Wood smoke contains dangerous toxins that cause changes in human cells. Because woodsmoke presents such a

high health risk, legislation is needed to regulate the use of open-air fires and wood-burning stoves.

Which of the following, if true, provides the most support for the argument above?

(A) The amount of dangerous toxins contained in woodsmoke is much less than the amount contained in an equal

volume of automobile exhaust.

(B) Within the jurisdiction covered by the proposed legislation, most heating and cooking is done with oil or

natural gas.

(C) Smoke produced by coal-burning stoves is significantly more toxic than smoke from wood-burning stoves.

(D) No significant beneficial effect on air quality would result if open-air fires were banned within the jurisdiction

covered by the proposed legislation.

(E) In valleys where wood is used as the primary heating fuel, the concentration of smoke results in poor air

quality.

100. The cost of producing radios in country Q is 10 percent less than the cost of producing radios in country

Y.Even after transportation fees and tariff charges are added, it is still cheaper for a company to import radios

from country Q to Country Y than to produce radios in Country Y.

Page 38: Critical Reasoning Questions - CATKing Educare · Critical Reasoning Questions . Borivali | Andheri |Powai & Pune Contact : 9029077114/ 8097607504 / 9619160261 / 8149774534 1. Homeowners

Borivali | Andheri |Powai & Pune

Contact : 9029077114/ 8097607504 / 9619160261 / 8149774534

www.catking.in

The statments above, if true, best support which of the following assertions?

A) Labor costs in country Q are 10 percent below those in Country Y.

B) Importing radios from country Q to country Y will eliminate 10 percent of the manufacturing jobs in Country Y.

C) The tariff on a radio imported from country Q to country Y is less than 10 percent of the cost of manufacturing

the radio in country Y

D) The fee for transporting a radio from country Q to Country Y is more than 10 percent of the cost of

manufacturing the radio in country Q

E) It takes 10 percent less time to manufacture a radio in country Q than it does in Country Y.

101. Two-dimensional bar codes are omni-directional; that is, unlike one-dimensional bar codes, they can be scanned

from any direction. Additionally, two-dimensional bar codes are smaller and can store more data than their one-

dimensional counterparts. Despite such advantages, two-dimensional bar codes account for a much smaller

portion of total barcode usage than one-dimensional barcodes.

Which of the following, if true, most helps to resolve the apparent paradox?

(A) Many smaller stores do not use bar codes at all because of the expense.

(B) For some products, the amount of data necessary to be coded is small enough to fit fully on a one-dimensional

bar code.

(C) Two-dimensional bar codes are, on average, less expensive than one-dimensional bar codes.

(D) Two-dimensional bar codes can also be scanned by consumer devices, such as cell phones.

(E) One-dimensional bar codes last longer and are less prone to error than two dimensional bar codes.

102. Traditionally, public school instructors have been compensated according to seniority. Recently, educational

experts have criticized the system as one that rewards lackadaisical teaching and reduces motivation to excel.

Instead, these

experts argue that, to retain exceptional teachers and maintain quality instruction, teachers should receive

salaries or bonuses based on performance ratherthan seniority.

Which of the following, if true, most weakens the argument of the educational

experts?

(A) Some teachers express that financial compensation is not the only factor contributing to job satisfaction and

teaching performance.

(B) School districts will develop their own unique compensation structures that may differ greatly from those of

other school districts.

(C) Upon leaving the teaching profession, many young, effective teachers cite a lack of opportunity for more rapid

financial advancement as a primary factor in the decision to change careers.

(D) In school districts that have implemented pay for performance compensation structures, standardized test

scores have dramatically increased.

Page 39: Critical Reasoning Questions - CATKing Educare · Critical Reasoning Questions . Borivali | Andheri |Powai & Pune Contact : 9029077114/ 8097607504 / 9619160261 / 8149774534 1. Homeowners

Borivali | Andheri |Powai & Pune

Contact : 9029077114/ 8097607504 / 9619160261 / 8149774534

www.catking.in

(E) A merit-based system that bases compensation on teacher performance reduces collaboration, which is an

integral component of quality instruction.

103. The national infrastructure for airport runways and air traffic control requires immediate expansion to

accommodate the increase in private, smaller planes. To help fund this expansion, the Federal Aviation Authority

(FAA) has proposed a fee for all air travelers. However, this fee would be unfair, as it would impose costs on all

travelers to benefit only the few who utilize the new private planes.

Which of the following, if true, would cast the most doubt on the claim that the

proposed fee would be unfair?

(A) The existing national airport infrastructure benefits all air travelers.

(B) The fee, if imposed, will have a negligible impact on the overall volume of air travel.

(C) The expansion would reduce the number of delayed flights resulting from small private planes congesting

runways.

(D) Travelers who use small private planes are almost uniformly wealthy or

traveling on business.

(E) A substantial fee would need to be imposed in order to pay for the expansion costs.

104. Student Advisor: One of our exchange students faced multiple arguments with her parents over the course of the past

year. Not surprisingly, her grade point average (GPA) over the same period showed a steep decline. This is just one

example of a general truth: problematic family relationships can cause significant academic difficulties for our

students. Which of the following is an assumption underlying the general truism claimed by the Student Advisor?

A. Last year, the exchange student reduced the amount of time spent on academic work, resulting in a lower GPA.

B. The decline in the GPA of the exchange student was not the reason for the student's arguments with her parents.

C. School GPA is an accurate measure of a student's intellectual ability.

D. If proper measures are not taken, the decline in the student's academic performance may become irreversible.

E. Fluctuations in academic performance are typical for many students.

105. Country X contains many rivers that flow down from its high mountains. These rivers have been dammed to harness

the hydroelectric power that can be derived from this resource. More than enough power is generated from these

dams to meet the country’s energy needs. Yet, citizens of Country X often experience power shortages or even

outages. Which of the following, if true, best explains the situation described above?

A. The flow of the rivers is heavier in the spring than at other times of year.

B. Citizens of Country X rely heavily on electronic appliances in their homes.

C. Country X has not invested in alternate sources of energy.

D. Most of the electricity generated in Country X is sold to other countries.

E. Some of the most powerful rivers in Country X have yet to be dammed.

106. As a result of consumers’ increased awareness of the health risks associated with heavy consumption of red meat,

the meat of the ostrich has become increasingly popular as a low-fat, low cholesterol alternative to beef. Accordingly,

the number of ostrich farms in the United States has nearly quadrupled since 1980, flooding markets with ostrich

Page 40: Critical Reasoning Questions - CATKing Educare · Critical Reasoning Questions . Borivali | Andheri |Powai & Pune Contact : 9029077114/ 8097607504 / 9619160261 / 8149774534 1. Homeowners

Borivali | Andheri |Powai & Pune

Contact : 9029077114/ 8097607504 / 9619160261 / 8149774534

www.catking.in

products. However, the price-per pound of ostrich meat, adjusted for inflation, has not decreased at all over the past

several years. Which of the following, if true, would best explain the steady price-per-pound of ostrich meat?

A. The demand for ostrich products has outpaced the supply.

B. Ostriches are especially difficult to breed in captivity.

C. Other types of meat, such as pork and poultry, have decreased in price over the past several years.

D. Purveyors of beef have not attempted to counter beef’s poor public image.

E. The number of supermarkets that carry ostrich meat has increased steadily since 1980.

107. A certain automaker aims to increase its market share by deeply discounting its vehicles' prices for the next

several months. The discounts will cut into profits, but because they will be heavily advertised the manufacturer

hopes that they will attract buyers away from rival manufacturers' cars. In the longer term, the automaker envisions

that customers initially attracted by the discounts may become loyal customers.

In assessing the plan's chances of achieving its aim, it would be most useful to know which of the following?

(A) Whether the automaker's competitors are likely to respond by offering deep discounts on their own products

(B) Whether the advertisements will be created by the manufacturer's current advertising agency

(C) Whether some of the automaker's models will be more deeply discounted than others

(D) Whether the automaker will be able to cut costs sufficiently to maintain profit margins even when the discounts

are in effect

(E) Whether an alternative strategy might enable the automaker to enhance its profitability while holding a constant

or diminishing share of the market

108. Company X receives most of its revenues from the sale of gasoline through a network of gas stations that it owns

across the country. The company purchases ready-for-sale gasoline from several oil refineries at wholesale prices and

sells the gasoline to the final consumer at its gas stations. Over the next quarter, the managers of Company X expect

that the market price of gasoline will rise by approximately 10 percent. Therefore, the managers project that the

next quarter’s revenues from the sale of gasoline will also increase by approximately 10 percent.

The managers’ projection is based on which of the following assumptions?

A) Consumption of gasoline at the company’s gas stations will not drop in response to higher prices.

B) Company profits will not decline below their current level.

C)Higher gasoline prices will not reduce the company’s revenues from other business lines.

D)The selling price of the company’s gasoline can be set at any price the managers decide is appropriate.

E)The supply of gasoline is likely to decline over the next quarter.

109. Outsourcing is the practice of obtaining from an independent supplier a product or service that a company has

previously provided for itself. Since a company’s chief objective is to realize the highest possible year-end profits,

any product or service that can be obtained from an independent supplier for less than it would cost the

company to provide the product or service on its own should be outsourced.

Which of the following, if true, most seriously weakens the argument?

(A) If a company decides to use independent suppliers for a product, it can generally exploit the vigorous

Page 41: Critical Reasoning Questions - CATKing Educare · Critical Reasoning Questions . Borivali | Andheri |Powai & Pune Contact : 9029077114/ 8097607504 / 9619160261 / 8149774534 1. Homeowners

Borivali | Andheri |Powai & Pune

Contact : 9029077114/ 8097607504 / 9619160261 / 8149774534

www.catking.in

competition arising among several firms that are interested in supplying that product.

(B) Successful outsourcing requires a company to provide its suppliers with information about its products and

plans that can fall into the hands of its competitors and give them a business advantage.

(C) Certain tasks, such as processing a company’s payroll, are commonly outsourced, whereas others, such as

handling the company’s core business, are not.

(D) For a company to provide a product or service for itself as efficiently as an independent supplier can provide

it, the managers involved need to be as expert in the area of that product or service as the people in charge

of that product or service at an independent supplier are.

(E) When a company decides to sue an independent supplier for a product or service, the independent supplier

sometimes hires members of the company’s staff who formerly made the product or provided the service

that the independent supplier now supplies.

110. Sports analyst: 90% of Country D's tennis professionals achieve positions in the international top ten, usually before

reaching the age of 20. It must be that Country D's population has genetic characteristics that are suited to playing

the sport of tennis.

Tennis coach: That's not true. Country D's success in tennis is not related to genetics, but to the skills and experience

of its players.

Which of the following, if true, most strengthens the tennis coach's objection to the analyst's claim?

A) The winner of last year's international tennis tour was a player from Country B.

B) The physique that a tennis player is born with is an extremely important factor that can affect that player's success

as a professional.

C) In Country D's education, tennis is a compulsory activity that is taught to students through all twelve years of their

formal education.

D) In the last Olympic Games, Country D did not win medals in any other discipline other than tennis.

E) Every year, 2% of Country D's tennis professionals are accused of using illegal substances to enhance their

performance.

111. A popular beach has long had a dolphin feeding program in which fish are given to dolphins several times a day; many

dolphins get as much as half of their food each day there. Although dolphins that first benefit from the program as

adults are healthy and long-lived, their offspring have a lower life expectancy than offspring of dolphins that feed

exclusively in the wild. Which of the following, if true, most helps to explain the lower life expectancy of offspring of

dolphins feeding at the beach compared to other young dolphins?

A. Sharks that prey on dolphins are less common in the open seas off the beach than in many other areas of the open

seas where dolphins congregate.

B. Many of the adult dolphins that feed at the beach are females that nurse their offspring there.

C. The fish given to the dolphins at the beach are the same types of fish that dolphins typically catch in the wild.

D. Many dolphins that feed at the beach with their offspring come to the beach only a few times a month.

E. Adult dolphins that feed at the beach spend much less time teaching their offspring how to catch fish in the wild

than do other adult dolphins.

Page 42: Critical Reasoning Questions - CATKing Educare · Critical Reasoning Questions . Borivali | Andheri |Powai & Pune Contact : 9029077114/ 8097607504 / 9619160261 / 8149774534 1. Homeowners

Borivali | Andheri |Powai & Pune

Contact : 9029077114/ 8097607504 / 9619160261 / 8149774534

www.catking.in

112. Because most hospitals suffer a chronic undersupply of physicians, patients must sometimes wait hours in the

emergency room to see a doctor. Nurses should therefore perform initial examinations in hospital emergency rooms to

determine which patients merit immediate treatment and which can wait until the emergency physicians have more

time to see them.

Which of the following is an assumption on which the argument above is based?

A) Hospitals should expand their medical staffs.

b) Physicians cannot be trained to perform initial examinations themselves.

c) Emergency rooms will run more smoothly if initial examinations are performed.

d) Hospitals are always fully staffed with nurses.

e) Nurses are competent to judge the severity of patients’ conditions.

113. The commissioner of a professional sports league dictated that teams could not put players on the field who had a

greater than 20 percent chance of suffering a career ending spinal injury during competition. The commissioner

justified this decision as a way to protect players from injury while protecting the league from lawsuits.

Which of the following, if true, would most undermine the effectiveness of the commissioner’s new policy?

A. Spinal injuries can result in paralysis, loss of fine motor skills, and even death.

B. The previous year, more than seven players in the league suffered career ending spinal injuries.

C. The players’ union agrees that the risk of injury is an inevitable part of playing the game at a professional level.

D. There is no scientifically valid method for determining the likelihood of any player suffering a career ending spinal

injury at any given time.

E. Players barred from playing because of this new regulation will be entitled to compensation for lost wages at a

level determined by the commissioner’s office.

114. A cost-effective solution to the problem of airport congestion is to provide high-speed ground transportation between

major cities lying 200 to 500 miles apart. The successful implementation of this plan would cost far less than

expanding existing airports and would also reduce the number of airplanes clogging both airports and airways.

Which of the following, if true, could proponents of the plan above most appropriately cite as a piece of evidence for

the soundness of their plan?

(A) An effective high-speed ground-transportation system would require major repairs to many highways and mass-

transit improvements.

(B) One-half of all departing flights in the nation’s busiest airport head for a destination in a major city 225 miles

away.

(C) The majority of travelers departing from rural airports are flying to destinations in cities over 600 miles away.

(D) Many new airports are being built in areas that are presently served by high-speed ground-transportation systems.

(E) A large proportion of air travelers are vacationers who are taking long-distance flights.

115. Phytonutrients, found in fresh fruits and vegetables, has beneficial effects on the human cardiovascular system.

Because heart disease is a major health concern, American doctors should encourage all people, especially those with

heart disease, to eat a diet high in fresh fruits and vegetables.

Page 43: Critical Reasoning Questions - CATKing Educare · Critical Reasoning Questions . Borivali | Andheri |Powai & Pune Contact : 9029077114/ 8097607504 / 9619160261 / 8149774534 1. Homeowners

Borivali | Andheri |Powai & Pune

Contact : 9029077114/ 8097607504 / 9619160261 / 8149774534

www.catking.in

Which of the following, if true, provides the most support for the argument above?

(A) the combination of soluble and insoluble fibers found in whole grains also support heart health

(B) a large percentage of those who suffer heart disease in the United States do not live close to large agricultural

area where fruits and vegetables are grown

(C) smoking cigarettes is a much more serious health risk than the lack of fresh fruits and vegetables in one's diet

(D) vegetarians tend to live longer than folks in the general population

(E) folks with heart disease who eat few or no fresh fruits and vegetables have heart attacks at a much higher rate

than do the folks who eat those foods.

116. In Washington County, attendance at the movies is just large enough for the cinema operators to make modest

profits. The size of the county's population is stable and is not expected to increase much. Yet there are investors

ready to double the number of movie screens in the county within five years, and they are predicting solid profits

both for themselves and for the established cinema operators.

Which of the following, if true about Washington County, most helps to provide a justification for the investors'

prediction?

(A) Over the next ten years, people in their teenage years, the prime moviegoing age, will be a rapidly growing

proportion of the county's population.

(B) As distinct from the existing cinemas, most of the cinemas being planned would be located in downtown areas, in

hopes of stimulating an economic revitalization of those areas.

(C) Spending on video purchases, as well as spending on video rentals, has been increasing modestly each year for the

past ten years.

(D) The average number of screens per cinema is lower among existing cinemas than it is among cinemas still in the

planning stages.

(E) The sale of snacks and drinks in cinemas accounts for a steadily growing share of most cinema operators' profits.

117. A certain pharmaceutical firm recently developed a new medicine, Dendadrine, which provides highly effective

treatment for severe stomach disorders that were previously thought to be incurable. However, to develop the new

medicine, the company spent nearly $5 billion in research and development costs. Given the size of the market for

Dendadrine and the amount of the initial investment in its development, the company would need to sell Dendadrine

at a price that is at least 5 times greater than its variable costs just to break even. Yet the company’s management

claims that Dendadrine will soon become the major driver of the firm’s profits ----Source CR 700 clubbers ---????

Which of the following statements best reconciles the management’s claim with the evidence on the expenditures

associated with the development of Dendadrine?

A. The pharmaceutical firm has been granted a patent to become the sole producer and distributor of Dendadrine,

and drugs under patent protection typically sell at prices that are at least 10 times greater than their variable costs.

B. Development of some pharmaceutical products involves substantial initial expenditures on research, testing, and

approval.

C. In issues related to personal health, corporate profits should not become the primary consideration.

D. Several other pharmaceutical companies are working on new medicines that may become effective substitutes for

Page 44: Critical Reasoning Questions - CATKing Educare · Critical Reasoning Questions . Borivali | Andheri |Powai & Pune Contact : 9029077114/ 8097607504 / 9619160261 / 8149774534 1. Homeowners

Borivali | Andheri |Powai & Pune

Contact : 9029077114/ 8097607504 / 9619160261 / 8149774534

www.catking.in

Dendadrine.

E. While Dendadrine can be highly effective in treating stomach disorders; it may also result in serious side effects

such as dizziness and hallucinations.

118. In virtually any industry, technological improvements increase labor productivity, which is the output of goods and

services per person-hour worked. In Parland's industries, labor productivity is significantly higher than it is in Vergia's

industries. Clearly, therefore, Parland's industries must, on the whole, be further advanced technologically than

Vergia's are.

The argument is most vulnerable to which of the following criticisms?

(A) It offers a conclusion that is no more than a paraphrase of one of the pieces of information provided in its support.

(B) It presents as evidence in support of a claim information that is inconsistent with other evidence presented in

support of the same claim.

(C) It takes one possible cause of a condition to be the actual cause of that condition without considering any other

possible causes.

(D) It takes a condition to be the effect of something that happened only after the condition already existed.

(E) It makes a distinction that presupposes the truth of the conclusion that is to be established.

119. Advanced Technology School and Computer Horizons provide training for information technology professionals. Both

schools spend fifty percent of their budget on professor salaries. In a new strategic initiative Computer

Horizons is attempting to gain higher profits than Advanced Technology. Computer Horizons should lower professor

salaries to accomplish this goal.

Which of the following, if true, would most weaken the argument above?

A. The remainder of Computer Horizons budget contains a large number of fixed costs.

B. Reducing professor salaries would lead to lower teaching quality which would subsequently cause a decrease in

student enrollment and tuition revenues.

C. Last year Advanced Technology replaced Computer Horizons as the number one ranked technology school in the

local newspaper's educational survey.

D. Computer Horizons professors earn fifteen percent more than their counterparts at Advanced Technology.

E. Computer Horizons and Advanced Technology are the sole information technology schools in their metropolis.

120. In the arid land along the Colorado River, use of the river's water is strictly controlled: farms along the river each

have a limited allocation that they are allowed to use for irrigation. But the trees that grow in narrow strips along the

river’s banks also use its water. Clearly, therefore, if farmers were to remove those trees, more water would be

available for crop irrigation

Which of the following, if true, most seriously weakens the argument?

a. The trees along the river’s banks shelter it from the sun and wind, thereby greatly reducing the amount of water

lost through evaporation

b. Owners of farms along the river will probably not undertake the expense of cutting down trees along the banks

unless they are granted a greater allocation of water in return

c. Many of the tree species currently found along the river’s banks are specifically adapted to growing in places where

Page 45: Critical Reasoning Questions - CATKing Educare · Critical Reasoning Questions . Borivali | Andheri |Powai & Pune Contact : 9029077114/ 8097607504 / 9619160261 / 8149774534 1. Homeowners

Borivali | Andheri |Powai & Pune

Contact : 9029077114/ 8097607504 / 9619160261 / 8149774534

www.catking.in

tree roots remain constantly wet.

d. The strip of land where trees grow along the river’s banks would not be suitable for growing crops if the trees were

removed.

e. The distribution of water allocations for irrigation is intended to prevent farms father upstream from using water

needed by farms father downstream

121. Exhaust released from a combustion engine is harmful when breathed in significant quantities. Since exhaust poses a

particular danger to health, Congress needs to place regulations on the use of motorized vehicles and lawn care tools.

Which of the following, if true, does the most to strengthen the given argument?

A In heavily populated cities where motorized vehicles are most common, the level of exhaust often exceeds

maximum safety levels.

B The health risks associated with exhaust are much less severe than those associated with an equivalent amount of

second hand cigarette smoke.

C The most commonly used mode of transportation in the population over which Congress has authority is bicycles.

D Coal burning locomotives produce a much deadlier exhaust than do combustion engines.

E If Congress were to ban lawn care tools, there would be no significant benefit to the health of the population over

which Congress has authority.

122. Mayor: Migrating shorebirds stop at our beach just to feed on horseshoe-crab eggs, a phenomenon that attracts

tourists. To bring more tourists, the town council plans to undertake a beach reclamation project to double the area

available to crabs for nesting.

Birdwatcher: Without a high density of crabs on a beach, migrating shorebirds will go hungry because shorebirds only

eat eggs that a crab happens to uncover when it is digging its own nest.

Which of the following, if true, would provide the mayor with the strongest counter to the birdwatcher’s objection?

A. Every year a certain percentage of crabs are caught by fishermen as bait for eel traps.

B. Horseshoe crabs are so prolific that given favorable circumstances their numbers increase rapidly.

C. On average, tourists who come to the town in order to watch birds spend more money there than tourists who

come for other purposes.

D. The additional land made available by the reclamation project will give migrating shorebirds more space.

Some of the migrating shorebirds make only one stop during their migration form South America to Canada

123. The difficulty with the proposed high-speed train line is that a used plane can be bought for one-third the price of

the train line, and the plane, which is just as fast, can fly anywhere. The train would be a fixed linear system, and we

live in a world that is spreading out in all directions and in which consumers choose the free-wheel systems (cars,

buses, aircraft), which do not have fixed routes. Thus a sufficient market for the train will not exist. to decrease in

the next few years.

Which of the following, if true, most severely weakens the argument presented above?

(A) Cars, buses, and planes require the efforts of drivers and pilots to guide them, whereas the train will be guided

Page 46: Critical Reasoning Questions - CATKing Educare · Critical Reasoning Questions . Borivali | Andheri |Powai & Pune Contact : 9029077114/ 8097607504 / 9619160261 / 8149774534 1. Homeowners

Borivali | Andheri |Powai & Pune

Contact : 9029077114/ 8097607504 / 9619160261 / 8149774534

www.catking.in

mechanically.

(B) Cars and buses are not nearly as fast as the high-speed train will be.

(C) Planes are not a free-wheel system because they can fly only between airports, which are less convenient for

consumers than the high-speed train's stations would be.

(D) The high-speed train line cannot use currently underutilized train stations in large cities.

(E) For long trips, most people prefer to fly rather than to take ground-level transportation.

124. The leading manufacturer of bottled sauces has announced that it intends to introduce a new line of sauces that will

be condensed by a factor of three to one. The condensed sauces will be marketed as pro-environment since the

manufacturer will be able to reduce the amount of packaging required for an equivalent amount of non-condensed

sauce. The manufacturer also claims that it will also be able to reduce the amount of packaging required to ship the

sauce, and correspondingly reduce the energy consumed to transport the sauce to supermarkets since the condensed

sauces will be lighter per unit. Many market analysts believe that the new condensed sauces could become more

popular than the older, non-condensed sauces.

Which one of the following, if true, most strongly supports the prediction made by the market analysts?

A. Consumers, in general, are guided by environmental considerations when making a purchase decision about a

sauce.

B. Supermarkets will give the condensed sauces the same degree of visibility on shelves as they do the traditional,

non-condensed sauces.

C. Some analysts estimate that consumers of the condensed sauces may end up paying more per unit of sauce

consumed than they would have for non-condensed sauces.

D. Consumer research shows that consumers might be initially reluctant to purchase the condensed sauces.

E. Other sauce manufacturers are unlikely to respond with their own condensed sauces any time soon.

125. A social worker surveyed 200 women who recently had given birth to their first child. Half of these women had

chosen to give birth in a hospital or obstetric clinic; the other half had chosen to give birth at home under the care of

certified MIdwives. Of the 100 births that occurred at home, only five had presented any substantial complications,

whereas 17 of the hospital births had required extra attention because of complications during delivery. The social

worker concluded from this survey that the home is actually a safer environment in which to give birth than is a

hospital or clinic.

Which of the following, if true, most seriously calls the social worker's conclusion above into question?

(A) All of the women in the study who were diagnosed as having a high possibility of delivery complications elected to

give birth in a hospital.

(B) Many obstetricians discourage their patients from giving birth in their own homes.

(C) Women who give birth in their own homes tend to experience less stress during labor and delivery than do those

who deliver in hospitals.

(D) Women who give birth in hospitals and clinics often have shorter periods of labor than do those who give birth at

home.

(E) Pregnant doctors prefer giving birth in a hospital.

Page 47: Critical Reasoning Questions - CATKing Educare · Critical Reasoning Questions . Borivali | Andheri |Powai & Pune Contact : 9029077114/ 8097607504 / 9619160261 / 8149774534 1. Homeowners

Borivali | Andheri |Powai & Pune

Contact : 9029077114/ 8097607504 / 9619160261 / 8149774534

www.catking.in

126. The climbing season of 2006 was the deadliest on record for those attempting to conquer Mount Everest, the

world’s tallest mountain, as more people perished attempting to reach the summit in 2006 than in any other year.

Interestingly, most mountaineering experts attribute the high number of fatalities, almost all of which occurred in

the unforgiving “death-zone” above 26,000 feet, directly to the exceptionally good weather prevalent during the

2006 climbing season. Which of the following, if true, best helps to explain the conclusion of the mountaineering

experts?

A) All of the forecasts were for extremely bad weather; the good weather was a significant surprise to all of the

climbers.

B) The good weather prompted significantly more people than ever to try to reach the summit and enter the

“deathzone,”

many of whom would have turned back at a lower altitude in poorer weather.

C) The good weather caused the “death-zone” to have warmer temperatures and less intense winds than in recent

years.

D)Modern equipment is particularly effective in protecting climbers from the elements in bad weather.

E) Many accomplished climbers don’t attempt Mt. Everest during good weather because they feel it is not a challenge.

127. In recent years, the Holdsville Transportation Authority (HTA) has noted consistent delays

on its Holdsville-River Valley bus, which runs from downtown Holdsville to the suburb of

River Valley. In order to decrease the commuting time from Holdsville to River Valley, the

HTA recently eliminated the James Street stop on the Holdsville-River Valley line.

However, data show that the average commuting time from Holdsville to River Valley has

actually increased since the elimination of the stop. Which of the following provides the best

explanation for the increase in commuting time from Holdsville to River Valley?

A) Almost all of the commuters who previously used the James Street stop now use the adjacent Green Street stop,

causing overcrowding and excessive boarding and de-boarding delays at the Green Street stop.

b) A small percentage of the commuters who previously used the James Street stop now use alternate modes of

transportation to commute from Holdsville to River Valley.

c) 90% of Holdsville-River Valley commuters were in favor of eliminating the James Street stop.

d) The Holdsville-River Valley bus route runs along River Street, which is always congested with heavy automobile

traffic.

e) The Johnstown bus line, another line operated by the HTA, has also experienced an increase in average commuting

time since eliminating a stop on its route.

128. Child psychologist: Children at school should be taught to simply ignore students who are taunting or bullying them.

In fact, children should make it a strict rule not to talk to anyone who has ever bullied or harassed them.

School Administrator: This is a terrible idea. If we implemented that plan, then none of the children in the school

would be talking to one another!

The school administrator’s response to the psychologist relies on which of the following assumptions?

(A) Students tend not to follow rules set up at the school.

(B) There is never a good reason for a student to bully another student.

(C) For any two students, at least one has bullied the other at some point.

Page 48: Critical Reasoning Questions - CATKing Educare · Critical Reasoning Questions . Borivali | Andheri |Powai & Pune Contact : 9029077114/ 8097607504 / 9619160261 / 8149774534 1. Homeowners

Borivali | Andheri |Powai & Pune

Contact : 9029077114/ 8097607504 / 9619160261 / 8149774534

www.catking.in

(D) There exists some way to prevent bullying.

(E) Bullying is the biggest problem among students at the school.

129. According to experts on shopping behavior, more shoppers would shop at Jerrod’s department store if they were

offered the convenience of shopping carts. In fact, even if the amount spent by these additional customers only just

covered the cost of providing the carts, providing carts would still probably increase Jerrod’s profits, since _______.

Which of the following most logically completes the argument?

A. the layout of Jerrod’s is open enough to accommodate shopping carts comfortably

B. several department stores that compete with Jerrod’s have begun to make shopping carts available to their

customers

C. there are some potential customers who would not be enticed to shop at Jerrod’s by the availability of shopping

carts

D. stores that make shopping carts available to customers usually have to hire people to retrieve them from parking

areas

E. a customer with a shopping cart buys more, on average, than a customer without a cart

130. Hunter: Many people blame hunters alone for the decline in Greenrock National Forest's deer population over

the past ten years. Yet clearly, black bears have also played an important role in this decline. In the past ten years,

the forest's protected black bear population has risen sharply, and examination of black bears found dead in the forest

during the deer hunting season showed that a number of them had recently fed on deer.

In the hunter's argument, the portion in boldface plays which of the following roles?

(A) It is the main conclusion of the argument.

(B) It is a finding that the argument seeks to explain.

(C) It is an explanation that the argument concludes is correct.

(D) It provides evidence in support of the main conclusion of the argument.

(E) It introduces a judgment that the argument opposes

131. A program instituted in a particular state allows parents to prepay their children’s future college

tuition at current rates. The program then pays the tuition annually for the child at any of the

state’s public colleges in which the child enrolls. Parents should participate in the program as a

means of decreasing the cost for their children’s college education.

Which of the following, if true, is the most appropriate reason for parents not to participate in

the program?

(A) The parents are unsure about which pubic college in the state the child will attend.

(B) The amount of money accumulated by putting the prepayment funds in an interest-bearing

account today will be greater than the total cost of tuition for any of the pubic colleges when the child enrolls.

(C) The annual cost of tuition at the state’s pubic colleges is expected to increase at a faster rate than the annual

increase in the cost of living

(D) Some of the state’s public colleges are contemplating large increases in tuition next year.

(E) The prepayment plan would not cover the cost of room and board at any of the state’s public colleges.

Page 49: Critical Reasoning Questions - CATKing Educare · Critical Reasoning Questions . Borivali | Andheri |Powai & Pune Contact : 9029077114/ 8097607504 / 9619160261 / 8149774534 1. Homeowners

Borivali | Andheri |Powai & Pune

Contact : 9029077114/ 8097607504 / 9619160261 / 8149774534

www.catking.in

132. Passengers must exit airplanes swiftly after accidents, since gases released following accidents are toxic to humans

and often explode soon after being released. In order to prevent passenger deaths from gas inhalation, safety officials

recommend that passengers be provided with smoke hoods that prevent inhalation of the gases.

Which of the following, if true, constitutes the strongest reason to require implementation of the safety officials'

recommendation?

(A) Test evacuations showed that putting on the smoke hoods added considerably to the overall time it took

passengers to leave the cabin.

(B) Some airlines are willing to buy the smoke hoods even though they consider them to be prohibitively expensive.

(C) Although the smoke hoods protect passengers from the toxic gases, they can do nothing to prevent the gases from

igniting.

(D) Some experienced flyers fail to pay attention to the safety instructions given on every commercial flight before

takeoff.

(E) In many airplane accidents, passengers who were able to reach emergency exits were overcome by toxic gases

before they could exit the airplane.

133. A large apartment building in a city was assessed by a city environmental bureau for the comparatively high

consumption of electricity in the building, and the landlord was threatened with a possible fine if the consumption

does not drop significantly in a specified period. While all built-in appliances and lighting in common areas is energy

efficient, the landlord found, upon doing an inspection, many residents were using older portable heaters that are not

very energy efficient. Many residents had a few of these heaters in different rooms of the apartment. The building is

often chilly, as the central heating system is old and in need of an upgrade, which will be tremendously expensive for

the landlord. The landlord decided to send each resident a detailed letter about the electric energy consumption of

these older portable heaters, recommending newer more energy efficient models that could be purchased at local

stores. The landlord hopes that enough residents will be motivated by this letter to buy the more energy efficient

heaters, so that he can avoid the fine.

Which of the following, if true, would provide most support for the prediction that the landlord's letter will have its

intended effect?

(A) Residents pay for their own monthly electrical use, and the rates in this city are high, so consuming less electricity

would result in a sizable monthly savings.

(B) Those apartments in the building with southern exposure get abundant sunlight, which has a substantial warming

effect, and many of those residents do not use portable heaters.

(C) Some cooking appliances, like a toaster or a waffle iron, consume considerably more electricity than a portable

heater during the time they are operating.

(D) Throughout the city, this same environmental agency threatened other landlord and business owners with fines for

various reasons, and in almost every case, the concerned parties were able to make sufficient changes to avoid the

fines.

(E) This landlord owns six apartment buildings of varying sizes in the city, and at each of the other five, the average

electrical consumption is at or below the city-wide average for buildings of that size.

Page 50: Critical Reasoning Questions - CATKing Educare · Critical Reasoning Questions . Borivali | Andheri |Powai & Pune Contact : 9029077114/ 8097607504 / 9619160261 / 8149774534 1. Homeowners

Borivali | Andheri |Powai & Pune

Contact : 9029077114/ 8097607504 / 9619160261 / 8149774534

www.catking.in

134. Installing scrubbers in smokestacks and switching to cleaner-burning fuel are the two methods available to Northern

Power for reducing harmful emissions from its plants. Scrubbers will reduce harmful emissions more than cleaner-

burning fuels will. Therefore, by installing scrubbers, Northern Power will be doing the most that can be done to

reduce harmful emissions from its plants.

Which of the following is an assumption on which the argument depends?

(A)Switching to cleaner-burning fuel will not be more expensive than installing scrubbers.

(B)Northern Power can choose from among various Kinds of scrubbers, some of which are more effective than others.

(C)Northern Power is not necessarily committed to reducing harmful emissions from its plants.

(D)Harmful emissions from Northern Power's plants cannot be reduced more by using both methods together than by

the installation of scrubbers alone.

(E)Aside from harmful emissions from the smokestacks of its plants, the activities of Northern Power do not cause

significant air pollution

135. Red blood cells in which the malarial-fever parasite resides are eliminated from a person’s body after 120 days.

Because the parasite cannot travel to a new generation of red blood cells, any fever that develops in a person more

than 120 days after that person has moved to a malaria-free region is not due to the malarial parasite.

Which of the following, if true, most seriously weakens the conclusion above?

(A) The fever caused by the malarial parasite may resemble the fever caused by flu viruses.

(B) The anopheles mosquito, which is the principal insect carrier of the malarial parasite, has been eradicated in

many parts of the world.

(C) Many malarial symptoms other than the fever, which can be suppressed with antimalarial medication, can

reappear within 120 days after the medication is discontinued.

(D) In some cases, the parasite that causes malarial fever travels to cells of the spleen, which are less frequently

eliminated from a person’s body than are red blood cells.

(E) In any region infested with malaria-carrying mosquitoes, there are individuals who appear to be immune to

malaria.

136. To reduce the danger to life and property posed by major earthquakes, scientists have been investigating several

techniques for giving advance warning of dangerous earthquakes. Since catfish swim erratically before earthquakes,

some investigators have proposed monitoring catfish to predict dangerous earthquakes.

Which of the following, if true, most seriously undermines the usefulness of the proposal?

(A) In Japan, which is subject to frequent earthquakes, the behavior of catfish has long been associated with

earthquakes.

(B) Mechanical methods for detecting earthquakes have not proved effective.

(C) Tremors lead to the release of hydrogen sulfide gas into water, thereby causing various fish and shellfish to behave

erratically.

(D) Careful construction can reduce the dangers posed by earthquakes.

(E) Even very slight, fleeting tremors cause catfish to swim erratically

137. In Europe, many large animal species, such as mammoths, became extinct soon after humans first migrated to the

animals' areas of habitation. The spread of such extinctions closely followed the pattern of human migration.

However, since humans tended to migrate to areas as the climate in those areas began to warm, the extinctions might

have been precipitated by the climatic warming rather than by human migration.

Page 51: Critical Reasoning Questions - CATKing Educare · Critical Reasoning Questions . Borivali | Andheri |Powai & Pune Contact : 9029077114/ 8097607504 / 9619160261 / 8149774534 1. Homeowners

Borivali | Andheri |Powai & Pune

Contact : 9029077114/ 8097607504 / 9619160261 / 8149774534

www.catking.in

Which of the following, if true, provides the best evidence that it was human migration and not climatic change that

precipitated the extinctions?

A. Many animal species, such as deer and bison, continued to thrive in Europe even after humans migrated there.

B. Several periods of marked climatic warming have occurred in Europe, during which many large animal species that

lived there became extinct.

C. Many animal species that became extinct in Europe survived longer in areas that were isolated from human

populations but that experienced intense climatic warming.

D. In some areas of Europe, only a few archaeological sites have yielded evidence that shows an overlap between the

arrival of humans and the extinction of large animals.

E. Some large animals had become extinct in Europe even before humans migrated there.

138. Even though most universities retain the royalties from faculty members’ inventions, the faculty members retain

the royalties from books and articles they write. Therefore, faculty members should retain the royalties from the

educational computer software they develop.

The conclusion above would be more reasonably drawn if which of the following were inserted into the argument as

an additional premise?

(A) Royalties from inventions are higher than royalties from educational software programs.

(B) Faculty members are more likely to produce educational software programs than inventions.

(C) Inventions bring more prestige to universities than do books and articles.

(D) In the experience of most universities, educational software programs are more marketable than are books and

articles.

(E) In terms of the criteria used to award royalties, educational software programs are more nearly comparable to

books and articles than to inventions.

139. The average life expectancy for the US population as a whole is 73.9 years, but children born in Hawaii will live an

average age of 77 years, and those born in Louisiana, 71.7 years. If a newly wed couple from Louisiana were to begin

their family in Hawaii, therefore, their children would be expected to live longer than would be the if the family

remained in Louisiana.

Which of the following, if true, would most seriously weaken the colclusion drawn in the passage?

A Insurance company statisticians do not belive that moving to Hawaii will significantly lengthen the average

Louisianan's life

B The governor of Louisiana has falsely alleged that statistics for his state are inaccurate.

C The longivity ascribed to Hawwai's current popilation is attributable mostly to genetically determined factors

D Thirty percent of all Louisianans can expect to live longer than 77 years

E Most of the Hawwaiian Islands have levels of air population well below the national average for the US"

140. The Eurasian ruffe, a fish species inadvertently introduced into North Americas Great Lakes in recent years, feeds on

the eggs of lake whitefish, a native species, thus threatening the lakes’ natural ecosystem. To help track the

ruffe’s spread, government agencies have produced wallet-sized cards about the ruffe. The cards contain pictures

Page 52: Critical Reasoning Questions - CATKing Educare · Critical Reasoning Questions . Borivali | Andheri |Powai & Pune Contact : 9029077114/ 8097607504 / 9619160261 / 8149774534 1. Homeowners

Borivali | Andheri |Powai & Pune

Contact : 9029077114/ 8097607504 / 9619160261 / 8149774534

www.catking.in

of the ruffe and explain the danger they pose; the cards also request anglers to report any ruffe they catch.

Which of the following, if true, would provide most support for the prediction that the agencies’ action will have

its intended effect?

A. The ruffe has spiny fins that make it unattractive as prey.

B. Ruffe generally feed at night, but most recreational fishing on the Great Lakes is done during daytime hours.

C. Most people who fish recreationally on the Great Lakes are interested in the preservation of the lake whitefish

because it is a highly prized game fish.

D. The ruffe is one of several nonnative species in the Great Lakes whose existence threatens the survival of lake

whitefish populations there.

E. The bait that most people use when fishing for whitefish on the Great Lakes is not attractive to ruffe.

141. While antibiotics have done inestimable good to humankind over the last seventy years, there are several drawbacks

to using antibiotics that, until recently, have been overlooked. The human microbiome, which consists of the trillions

of bacteria that reside in each person’s body, is essential to good health. Specifically, the body contains and requires

both “good” and “bad” bacteria. It is when the proper equilibrium between the “good” bacteria and the “bad”

bacteria is disrupted that a number of health issues can emerge. Nonetheless, antibiotics indiscriminately kill both the

“good” and the “bad” bacteria, so each course of antibiotics should be followed by a treatment that

__________________________________

Which of the following most logically completes the argument above?

A. bypasses the microbiome altogether

B. targets and eliminates only the “bad” bacteria

C. restores the correct balance between the “good” and “bad” bacteria

D. purges the body of the remaining antibiotics completely

E. reverses any effects from the antibiotics

142. An advertising agency must leave its office building. To save overhead costs, it is considering setting up a "virtual

office" instead of moving to an adjacent office building. Employees will do their work on computers, as they do now,

but will do so at home, sharing their work with colleagues by communicating text and designs to colleagues'

computers through telephone lines, receiving typed comments, and then revising the work until it is satisfactory.

Which of the following, if true, indicates a disadvantage of the virtual office plan?

A. The agency can move its computers and associated equipment into employees' homes without purchasing any new

equipment or requiring employees to purchase such equipment.

B. To reach the current office building and return home, employees spend an average of one and a half hours per day

commuting, usually in heavy traffic.

C. The employees of the advertising agency are accustomed to autonomy in monitoring their own progress and

productivity.

D. Employees would be able to adapt quickly to using computers in a virtual office setting, since employees' current

use of computers to produce designs requires a high level of skill.

E. Expressions and gestures in face-to-face exchanges are an important aid to employees in gauging the viability of

new ideas they have proposed.

Page 53: Critical Reasoning Questions - CATKing Educare · Critical Reasoning Questions . Borivali | Andheri |Powai & Pune Contact : 9029077114/ 8097607504 / 9619160261 / 8149774534 1. Homeowners

Borivali | Andheri |Powai & Pune

Contact : 9029077114/ 8097607504 / 9619160261 / 8149774534

www.catking.in

143. Because no employee wants to be associated with bad news in the eyes of a superior, information about serious

problems at lower levels is progressively softened and distorted as it goes up each step in the management hierarchy.

the chief executive is therefore, less well informed about problems at lower levels than are his or her subordinates at

those levels.

The conclusion drawn above is based on the assumption that

a) problems should be solved at levels in the management hierarchy at which they occur

b) employees should be rewarded for accurately reporting problems to their superiors

c) problems-solving ability is more important at higher levels than it is at lower levels of the management hierarchy

d) chief executives obtain information about problems at lower levels from the no source other than their

subordinates

e) some employees are more are concerned about truth than about the way they are perceived by their superiors

144. A large pharmaceutical company has recently developed an antibiotic that has been shown to be highly effective at

treating a variety of infections. The drug is obtained from the scales of the rare tutu fish, which is only found in the

wild living in the reefs off Sando Island in the South Pacific. Since it takes the scales from 1,000 fish to make a single

kilogram of the antibiotic, it follows that continued production of the drug will lead to the extinction of the tutu fish.

Which of the following, if true, most seriously weakens the argument above?

(A) The antibiotic made from the tutu fish's scales is only available by prescription.

(B) Because tutu fish are rare, they are also expensive.

(C) The inhabitants of Sando Island regard the meat of the tutu fish as a delicacy.

(D) Tutu fish breed well in captivity, and can be cultivated on fish farms.

(E) Sando Island is difficult to reach since it does not have an airstrip.

145. According to futuristic writings in the 1960s, robots would soon drastically reduce crime. With night vision and ability

to detect the chemicals involved in ballistics, such robots could be programed to paralyze anyone roaming the street

at night with a gun: virtually all criminals fit that description. These criminals would be incapacitated and thus unable

to resist an easy arrest.

Which of the following, if true, most strongly indicates that the logic of the prediction is flawed?

1. Such robots would need to be charged during the daytime.

2. Since policemen carry guns, the robots would incapacitate them just as efficiently as they incapacitate criminals

3. Because these robots could pose a hazard to cars at night, special barriers would have to be constructed between

the paths of the robots and the lanes of traffic.

4. It's not obvious that reducing the number of criminals will always be beneficial.

5. If the proposal plan were successful, it might ultimate result in a smaller and more efficient police force.

146. Children of alcoholics are much more likely to become alcoholics than are children of nonalcoholics. The likelihood

varies depending on which parent is alcoholic and is much greater if both are alcoholic. This pattern holds true even if

children are separated from their alcoholic parent[s] at birth.

Page 54: Critical Reasoning Questions - CATKing Educare · Critical Reasoning Questions . Borivali | Andheri |Powai & Pune Contact : 9029077114/ 8097607504 / 9619160261 / 8149774534 1. Homeowners

Borivali | Andheri |Powai & Pune

Contact : 9029077114/ 8097607504 / 9619160261 / 8149774534

www.catking.in

Which of the following conclusions can most reliably be drawn from the statements above?

A) Children of nonalcoholics do not become alcoholics.

B) Fathers influence their children’s alcoholic tendencies more than mothers do.

C) A tendency toward alcoholism can be at least partially determined by genetics.

D) Environment has little influence on alcoholism.

E) Because it is inherited, alcoholism is difficult to treat.

147. The technological conservatism of bicycle manaufacturers is a reflection of the kinds of demand they are trying to

meet. The only cyclist seriously interested in innovation and willing to pay for it are bicycle racers. Therefore,

innovation in bicycle technology is limited by what authorities will accept as standard for purpose of competition in

bicycle races.

Which of following is an assumption made in drawing the conclusion above?

a) The market for cheap, traditional bicycle cannnot expand unless the market for high-performance competition

bicycles expands.

b) High-performance bicycles are likely to be improved more as a result of technological innovations developed in

small workshops than as a result of technological innovations developed in major manufacturing concerns.

c) Bicycle racers do not generate a strong demand for innovations that fall outside what is officially recoginized as

standard for purpose of competition.

d) The technology conservatism of bicycle manufacturers results primarily from their desire to manufacturer a product

that can be sold without being altered to suit different national markets.

e) The authorities who set standards for high-performance bicycle racing do not keep informed about innovative

bicycle design.

148. According to futuristic writings in the 1960s, robots would soon drastically reduce crime. With night vision and ability

to detect the chemicals involved in ballistics, such robots could be programed to paralyze anyone roaming the street

at night with a gun: virtually all criminals fit that description. These criminals would be incapacitated and thus unable

to resist an easy arrest.

Which of the following, if true, most strongly indicates that the logic of the prediction is flawed?

(A) Such robots would need to be charged during the daytime.

(B) Since policemen carry guns, the robots would incapacitate them just as efficiently as they incapacitate criminals

(C) Because these robots could pose a hazard to cars at night, special barriers would have to be constructed between

the paths of the robots and the lanes of traffic.

(D) It's not obvious that reducing the number of criminals will always be beneficial.

(E) If the proposal plan were successful, it might ultimate result in a smaller and more efficient police force.

149. When limitations were in effect on nuclear-arms testing, people tended to save more of their money, but when

nuclear-arms testing increased, people tended to spend more of their money. The perceived threat of nuclear

catastrophe, therefore, decreases the willingness of people to postpone consumption for the sake of saving money.

The argument above assumes that

Page 55: Critical Reasoning Questions - CATKing Educare · Critical Reasoning Questions . Borivali | Andheri |Powai & Pune Contact : 9029077114/ 8097607504 / 9619160261 / 8149774534 1. Homeowners

Borivali | Andheri |Powai & Pune

Contact : 9029077114/ 8097607504 / 9619160261 / 8149774534

www.catking.in

A) the perceived threat of nuclear catastrophe has increased over the years

B) most people supported the development of nuclear arms

C) people’s perception of the threat of nuclear catastrophe depends on the amount of nuclear-arms testing being

done

D) the people who saved the most money when nuclear-arms testing was limited were the ones who supported such

limitations

E) there are more consumer goods available when nuclear-arms testing increases

150. When limitations were in effect on nuclear-arms testing, people tended to save more of their money, but when

nuclear-arms testing increased, people tended to spend more of their money. The perceived threat of nuclear

catastrophe, therefore, decreases the willingness of people to postpone consumption for the sake of saving money.

The argument above assumes that

A) the perceived threat of nuclear catastrophe has increased over the years

B) most people supported the development of nuclear arms

C) people’s perception of the threat of nuclear catastrophe depends on the amount of nuclear-arms testing being

done

D) the people who saved the most money when nuclear-arms testing was limited were the ones who supported such

limitations

E) there are more consumer goods available when nuclear-arms testing increases

151. If the county continues to collect residential trash at current levels, landfills will soon be overflowing and parkland

will need to be used in order to create more space. Charging each household a fee for each pound of trash it puts out

for collection will induce residents to reduce the amount of trash they create; this charge will therefore protect the

remaining county parkland.

Which of the following is an assumption made in drawing the conclusion above?

(A) Residents will reduce the amount of trash they put out for collection by reducing the number of products they

buy.

(B) The collection fee will not significantly affect the purchasing power of most residents, even if their households do

not reduce the amount of trash they put out.

(C) The collection fee will not induce residents to dump their trash in the parklands illegally.

(D) The beauty of county parkland is an important issue for most of the county’s residents.

(E) Landfills outside the county’s borders could be used as dumping sites for the county’s trash.

152. Traditionally, decision-making by managers that is reasoned step-by-step has been considered preferable to

intuitive decision-making. However, a recent study found that top managers used intuition significantly more

than did most middle- or lower-level managers. This confirms the alternative view that intuition is actually more

effective than careful, methodical reasoning.

The conclusion above is based on which of the following assumptions?

(A) Methodical, step-by-step reasoning is inappropriate for making many real-life management decisions.

(B) Top managers have the ability to use either intuitive reasoning or methodical, step-by-step reasoning in making

Page 56: Critical Reasoning Questions - CATKing Educare · Critical Reasoning Questions . Borivali | Andheri |Powai & Pune Contact : 9029077114/ 8097607504 / 9619160261 / 8149774534 1. Homeowners

Borivali | Andheri |Powai & Pune

Contact : 9029077114/ 8097607504 / 9619160261 / 8149774534

www.catking.in

decisions.

(C) The decisions made by middle- and lower-level managers can be made as easily by using methodical reasoning as

by using intuitive reasoning.

(D) Top managers use intuitive reasoning in making the majority of their decisions.

(E) Top managers are more effective at decision-making than middle- or lower-level managers.

153. Bank depositors in the United States are all financially protected against bank failure because the government

insures all individuals' bank deposits. An economist argues that this insurance is partly responsible for the high rate of

bank failures, since it removes from depositors any financial incentive to find out whether the bank that holds their

money is secure against failure. If depositors were more selective, then banks would need to be secure in order to

compete for depositors' money.

The economist's argument makes which of the following assumptions?

(A) Bank failures are caused when big borrowers default on loan repayments.

(B) A significant proportion of depositors maintain accounts at several different banks.

(C) The more a depositor has to deposit, the more careful he or she tends to be in selecting a bank.

(D) The difference in the interest rates paid to depositors by different banks is not a significant factor in bank

failures.

(E) Potential depositors are able to determine which banks are secure against failure.

154. Traditionally, decision-making by managers that is reasoned step-by-step has been considered preferable to intuitive

decision-making. However, a recent study found that top managers used intuition significantly more than did most

middle- or lower-level managers. This confirms the alternative view that intuition is actually more effective than

careful, methodical reasoning.

The conclusion above is based on which of the following assumptions?

(A) Methodical, step-by-step reasoning is inappropriate for making many real-life management decisions.

(B) Top managers have the ability to use either intuitive reasoning or methodical, step-by-step reasoning in making

decisions.

(C) The decisions made by middle- and lower-level managers can be made as easily by using methodical reasoning as

by using intuitive reasoning.

(D) Top managers use intuitive reasoning in making the majority of their decisions.

(E) Top managers are more effective at decision-making than middle- or lower-level managers.

155. The downturn in the economy last year has prompted many companies to make widely publicized layoffs, resulting in

thousands of lost jobs. Economists predicted that these layoffs would cause people generally to cut back on their

discretionary spending even if their jobs were secure, in anticipation of coming hard times. However, this prediction

has not come to pass, since there has been no increase in the amount of money set aside by the general public in

savings accounts. The answer to which of the following questions would be most useful in evaluating the significance

of the savings patterns described above?

A. What business sectors were most affected by the layoffs?

Page 57: Critical Reasoning Questions - CATKing Educare · Critical Reasoning Questions . Borivali | Andheri |Powai & Pune Contact : 9029077114/ 8097607504 / 9619160261 / 8149774534 1. Homeowners

Borivali | Andheri |Powai & Pune

Contact : 9029077114/ 8097607504 / 9619160261 / 8149774534

www.catking.in

B. How much of their savings, on average, do laid-off employees deplete before finding new employment?

C. What has been the percent increase in the cost of necessities such as food, housing, and utilities during the period

since the layoffs?

D. What percentage of people laid off have savings accounts?

E. What has been the average salary during the period since the layoffs?

156. To decrease the number of crimes in city Y, the city’s Police Commissioner proposed taking some police officers from

low-crime districts of the city and moving them to high-crime districts of the city. His proposal is based on city Y

crime data that show that the number of crimes in any district of the city decreases when additional police officers

are moved into that district. The Police Commissioner’s proposal depends on which of the following assumptions?

A) City X experienced a drastic reduction in crime after implementing a proposal similar to that proposed by the

Police Commissioner of city Y.

B) The severity of crimes committed in any district of the city decreases when additional police officers are moved

into that district.

C) The number of crimes committed in all high-crime districts of city Y is more than triple the number of crimes

committed in all low-crime districts of city Y.

D) There are more low-crime districts than high-crime districts in city Y.

E) Districts of the city from which police officers are removed do not experience significant crime increases shortly

after the removal of those officers.

157. Violent forms of robbery tend to be most severe in nations with median wages not in the bottom 10% of nations,

but rather between the 10% and 20% intervals. Although that shifted bulge might seem to run counter to what

many might presume would be the case, there is a reasonable explanation for the bulge effect. As an economy

begins develop, an initial wave of envy tends to trigger robberies, including violent robberies. As the economy

develops further, a greater percentage of that society exits poverty, which in turn reduces envy, and thereby the

number of robberies. Therefore, we can expect Baltria to soon see its violent robbery rate decrease given that its

economy has begun to develop over the last several years.

Which one of the following is an assumption on which the argument depends?

A) Once an economy begins to develop, there tends to be a tightening of law enforcement, and punishment for

violent crime offenses.

B) Once an economy develops, a spill-off effect can help bolster the economies of neighboring countries, further

reinforcing economic development.

C) Baltria's economic development will not stall in the near-future.

D) Baltrian leaders are committed to making Baltria one of the most economically advanced nations in the region.

E) No other country in the region has a violent robbery rate as high as that of Baltria’s.

158. This year, the Rocktown school district offered a free summer enrichment program called "History Rocks" to its rising

fourth-grade students. Therefore, fourth graders in the Rocktown school district will score better this year on

American history test

Page 58: Critical Reasoning Questions - CATKing Educare · Critical Reasoning Questions . Borivali | Andheri |Powai & Pune Contact : 9029077114/ 8097607504 / 9619160261 / 8149774534 1. Homeowners

Borivali | Andheri |Powai & Pune

Contact : 9029077114/ 8097607504 / 9619160261 / 8149774534

www.catking.in

Which of the following, if true, would NOT strengthen the author's conclusion?

A - "History Rocks" focuses entirely on American history.

B- The majority of the district's rising fourth graders attended the program.

C- The district decided to make the program free because many students in the area come from low-income families.

D- The material on fourth-grade history tests in Rocktown is substantially similar to the material being covered in the

enrichment program.

E- It has been proven that students retain knowledge better when learning one subject at a time, as is the case in the

"History Rocks" program.

159. A convenience store manager noticed that a cooler which had been stocked with only a cola product and an iced-tea

product had 15 colas left at the end of the day but only 3 iced-tea beverages. As a result, the manager reasoned that

he should increase the amount of iced tea and decrease the amount of cola he ordered from the distributor.

Which of the following, if true, would most strengthen the manager's rationale for ordering more iced tea and less

cola?

a. The cooler in question is the only place in the store where the cola and iced tea beverages are stocked.

b. On that day, a month-long $1,000,000 sweepstakes began, with prizes awarded via the bottlecaps on the iced tea

beverage.

c. At the beginning of the day, the cooler was stocked with at least as many of the iced tea beverages as of the cola

beverages.

d. On the subsequent day, the remaining three iced tea beverages all sold within the first hour after the store

opened.

e. During that week, a special "buy one, get one free" sale was in effect for the cola beverage.

160. In a recent poll, 71% of respondents reported that they cast votes in the most recent national election. Voting

records show, however, that only 60% of eligible voters actually voted in that election.

Which of the following pieces of evidence, if true, would provide the best explanation for the discrepancy?

(A) The margin of error for the survey was plus or minus five percentage points.

(B) Fifteen percent of the surveys respondents were living overseas at the time of the election.

(C) Prior research has shown that people who actually do vote are also more likely to respond to polls than those who

do not vote.

(D) Some people who intend to vote are prevented from doing so by last minute conflicts on election day or other

complications.

(E) Polls about voting behavior typically have margins of error within plus or minus three percentage points.

161. Mechanicorp’s newest product costs so little to make that it appears doubtful the company will be able to sell it

without increasing the markup the company usually allows for profit: potential clients would simply not believe that

something so inexpensive would really work. Yet Mechanicorp’s reputation is built on fair prices incorporating only

modest profit margins.

The statements above, if true, most strongly support which of the following?

(A) Mechanicorp will encounter difficulties in trying to set a price for its newest product that will promote sales

without threatening to compromise the company’s reputation.

(B) Mechanicorp achieves large annual profits, despite small profits per unit sold, by means of a high volume of sales.

Page 59: Critical Reasoning Questions - CATKing Educare · Critical Reasoning Questions . Borivali | Andheri |Powai & Pune Contact : 9029077114/ 8097607504 / 9619160261 / 8149774534 1. Homeowners

Borivali | Andheri |Powai & Pune

Contact : 9029077114/ 8097607504 / 9619160261 / 8149774534

www.catking.in

(C) Mechanicorp made a significant computational error in calculating the production costs for its newest product.

(D) Mechanicorp’s newest product is intended to perform tasks that can be performed by other devices costing less to

manufacture.

(E) Mechanicorp’s production processes are designed with the same ingenuity as are the products that the company

makes.

162. The head baker at Barry’s Bagels can either purchase flour in-person from the local flour mill, Larry’s Local Mill, or

order a shipment of flour from an out-of-state mill, Isadore’s

Interstate Mill. The cost of the flour from Isadore’s Interstate Mill is 10 percent less than the cost of the flour from

Larry’s Local Mill. Even after shipping and handling fees are added, it

is still cheaper to order flour that has to be shipped from Isadore’s than to buy flour locally from Larry’s. The

statements above, if true, best support which of the following assertions?

A) Production costs at Isadore’s Interstate Mill are 10 percent below those at Larry’s Local Mill.

b) Buying flour from Isadore’s Interstate Mill will eliminate 10 percent of the local flour mill jobs.

c) The shipping and handling fees for a batch of flour purchased from Isadore’s Interstate Mill are less than 10 percent

of the cost of an identical batch of flour purchased from Larry’s

Local Mill.

d) The shipping and handling fees for a batch of flour purchased from Isadore’s Interstate Mill are more than 10

percent of the cost of Isadore’s flour.

e) Isadore’s Interstate Mill produces flour 10% more efficiently than Larry’s Local Mill does.

163. If there is an oil-supply disruption resulting in higher international oil prices, domestic oil prices in open-market

countries such as the United States will rise as well, whether such countries import all or none of their oil

Which of the following conclusions is best supported by the statement above?

(A) Domestic producers of oil in open-market countries are excluded from the international oil market when there is a

disruption in the international oil supply.

(B) International oil-supply disruptions have little, if any, effect on the price of domestic oil as long as an open-

market country has domestic supplies capable of meeting domestic demand.

(C) The oil market in an open-market country is actually part of the international oil market, even if most of that

country’s domestic oil is usually sold to consumers within its borders.

(D) Open-market countries that export little or none of their oil can maintain stable domestic oil prices even when

international oil prices rise sharply.

(E) If international oil prices rise, domestic distributors of oil in open-market countries will begin to import more oil

than they export.

164. Students who perform average work in school sometimes receive higher grades than they actually deserve. These

students are able to convince teachers that they truly enjoy the subject matter, are stimulated by the lectures, or

have a personal interest in the instructor. These students must be careful, however, because exaggerated efforts may

backfire.

If the statements above are true, which of the following draws the most reliable conclusion from the paragraph?

Page 60: Critical Reasoning Questions - CATKing Educare · Critical Reasoning Questions . Borivali | Andheri |Powai & Pune Contact : 9029077114/ 8097607504 / 9619160261 / 8149774534 1. Homeowners

Borivali | Andheri |Powai & Pune

Contact : 9029077114/ 8097607504 / 9619160261 / 8149774534

www.catking.in

(A) Students are dishonest when it comes to grades.

(B) Teachers may give grades to students for reasons other than classroom performance.

(C) Good students will seldom resort to trickery in achieving good grades.

(D) Teachers want to help their students to excel in the classroom.

(E) The perceptions of average students are important in achieving good grades.

165. Not all stars emit energy as visible light. Some stars emit energy at high frequencies outside the realm of visible

light, such as gamma rays. Other stars, particularly in the Milky Way galaxy, will emit energy in the form of invisible

X-rays.

If the statements above are true, which of the following conclusions is best supported?

A. A thorough analysis of visible light emissions is insufficient to monitor star energy emissions.

B. Gamma rays are also a common type of energy emission in the Milky Way galaxy.

C. X-rays emissions found outside the Milky Way galaxy are always paired with visible light emissions from the same

star.

D. Dying stars are more likely to emit gamma rays.

E. Telescopes capable of observing gamma rays are incapable of observing visible light frequencies.

166. Both enrollment and total tuition revenue at Brownsville University have increased during each of the last four years.

During the same period of time, enrollment at Canterbury University has steadily decreased, while total tuition

revenue has remained constant.

Which of the following assertions is best supported by the statements above?

A) Brownsville University now collects more total tuition revenue than Canterbury University.

(B) Students regard higher tuition as an indicator of higher quality.

(C) The per-student tuition at Canterbury University has risen over the last four years.

(D) Within four years, enrollment at Brownsville University will likely exceed enrollment at

Canterbury University.

(E) Canterbury University will likely continue to raise tuition to make up for lost revenue from

declining enrollment.

167. Interviewer: An alarming statistic reported in the Hobern Medical Journal is that 90 percent of the people in this

country now report that they know someone who has heart disease.

Dr. Summer: But an expected level of heart disease is 5 percent, or in other words, 1 out of every 20 people. So at

any given time if a person knows approximately 50 people, 1 or more will very likely suffer from heart disease.

Dr. Summer’s argument is structured to lead to which of the following conclusion?

A. The fact that 90 percent of the people know someone who suffers from heart disease is not an indication that heart

disease is abnormally high.

B. The current level of heart disease is not moderate.

C. If at least 5 percent of the population suffered from heart disease, the result of questioning a representative group

of people cannot be the percentage the interviewer cites.

Page 61: Critical Reasoning Questions - CATKing Educare · Critical Reasoning Questions . Borivali | Andheri |Powai & Pune Contact : 9029077114/ 8097607504 / 9619160261 / 8149774534 1. Homeowners

Borivali | Andheri |Powai & Pune

Contact : 9029077114/ 8097607504 / 9619160261 / 8149774534

www.catking.in

D. It is unlikely that the people whose statements the interviewer cites are giving accurate reports.

E. If a person with heart disease is given as a certain percent, the actual percentage of those with heart disease is

even higher.

168. Although most smoking-related illnesses are caused by inhaling the tar in tobacco smoke, it is addiction to nicotine

that prevents most smokers from quitting. In an effort to decrease the incidence of smoking-related illnesses,

lawmakers in Sandonia plan to reduce the average quantity of nicotine per cigarette by half over the next five years.

Unfortunately, smokers who are already addicted to nicotine tend to react to such reductions by smoking

correspondingly more cigarettes.

The information above most strongly supports which of the following predictions about the effects of implementing

the Sandonian government’s plan?

A. The average quantity of tar inhaled by Sandonian smokers who are currently addicted to nicotine will probably not

decrease during the next five years.

B. Sandonian smokers who are not already addicted to nicotine will probably also begin to smoke more cigarettes

during the next five years than they had previously.

C. The annual number of Sandonian smokers developing smoking-related illnesses will probably decrease during the

next five years

D. The proportion of Sandonians attempting to quit smoking who succeed in that attempt will probably decrease

during the next five years.

E. The number of Sandonians who quit smoking during the next five years will probably exceed the number who quit

during the last five years.

169. A law that is not consistently enforced does not serve its purpose. Law without enforcement is not law, it is merely

stature-a promise of law. To institute real law is not merely to declare that such and such behaviour is forbidden, it is

also to punish those who violate that edict. Furthermore, those who enforce law must punish without favour for their

friends or malice for their enemies. To punish only those own dislike while forgiving others is not to enforce law but to

engage in the arbitrary and unjust exercise of power.

The main point of the passage is that instituting real law consists in

(A) the exercise of power

(B) authorizing the enforcement of punishments

(C) the unbiased punishment of prohibited behaviour

(D) understanding the purpose of law

(E) clearly defining unacceptable behavior

170. Scientific research that involves international collaboration has produced papers of greater influence, as measured

by the number of times a paper is cited in subsequent papers, than has research without any collaboration. Papers

that result from international collaboration are cited an average of seven times, whereas papers with single authors

are cited only three times on average. This difference shows that research projects conducted by international

research teams are of greater importance than those conducted by single researchers.

Which one of the following is an assumption on which the argument depends?

Page 62: Critical Reasoning Questions - CATKing Educare · Critical Reasoning Questions . Borivali | Andheri |Powai & Pune Contact : 9029077114/ 8097607504 / 9619160261 / 8149774534 1. Homeowners

Borivali | Andheri |Powai & Pune

Contact : 9029077114/ 8097607504 / 9619160261 / 8149774534

www.catking.in

(A) Prolific writers can inflate the number of citations they receive by citing themselves in subsequent papers.

(B) It is possible to ascertain whether or not a paper is the product of international collaboration by determining the

number of citations it has received

(C) The number of citations a paper receives is a measure of the importance of the research it reports.

(D) The collaborative efforts of scientists who are citizens of the same country do not produce papers that are as

important as papers that are produced by international collaboration.

(E) International research teams tend to be more generously funded than are single researchers.

171. Several movie versions of Charles Dickens' Tale of Two Cities have been made. The original movie version made in

1939 is the best because it is closest in spirit to the original novel.

An underlying assumption of the argument above is that a movie based on a novel should:

A) reflect the director's original interpretation of the main themes of the novel.

B) accurately depict the time and place in which the novel is set.

C) feature actors and actresses who closely resemble the characters in the novel both in body and spirit.

D) faithfully render the details of the plot from the narrator's point of view.

E) capture the true meaning and intention of the novel.

172. Studies of fatal automobile accidents reveal that, in the majority of cases in which one occupant of an automobile is

killed while another survives, it is the passenger, not the driver, who is killed. It is ironic that the innocent passenger

should suffer for the driver's carelessness, while the driver often suffers only minor injuries or none at all.

Which of the following is an assumption underlying the reasoning in the passage above?

A) In most fatal automobile accidents, the driver of a car in which an occupant is killed is at fault.

B) Drivers of automobiles are rarely killed in auto accidents.

C) Most deaths in fatal automobile accidents are suffered by occupants of cars rather than by pedestrians.

D) Auto safety experts should increase their efforts to provide protection for those in the passenger seats of

automobiles.

E) Automobile passengers sometimes play a contributing role in causing auto accidents.

173. One of the truisms of the advertising industry is that it is rarely necessary to say something of substance in an

advertisement in order to boost sales. Instead, one only needs to attract the potential customer’s attention; memory

does the rest, for it is more important for sales that people know of a product than that they know something about

it.

Which of the following is assumed by the argument?

A. People can remember a product without having much information about it.

B. Advertisements, in their own way, function to improve people’s memories.

C. Attracting a potential customer’s attention is a simple matter.

D. The advertising industry knows little of substance about the products it promotes.

E. Advertisements seldom tell the truth about a product.

174. Military Consultant: The chain of command in your unit is marred by a serious flaw: your soldiers are so fearful of

being disciplined for security breaches within their jurisdiction that they fail to report potential problems to their

superiors. And those superiors share the same fear of being seen as lacking control of their units that they fail to fully

Page 63: Critical Reasoning Questions - CATKing Educare · Critical Reasoning Questions . Borivali | Andheri |Powai & Pune Contact : 9029077114/ 8097607504 / 9619160261 / 8149774534 1. Homeowners

Borivali | Andheri |Powai & Pune

Contact : 9029077114/ 8097607504 / 9619160261 / 8149774534

www.catking.in

investigate potential problems and bring them up the chain to you. Consequently, General, you’re likely presiding

over several security threats that you’ll never know about.

Which of the following is an assumption required by the consultant’s logic.

A. The general is responsible for the culture that exists within his chain of command.

B. All soldiers near the bottom of the chain of command are fearful of being disciplined for security breaches.

C. The general does not have sources other than the chain of command to alert him to security concerns.

D. Some soldiers do not fear authority figures more than they fear genuine security threats.

E. There is not a feasible incentive system to reward soldiers for conscientiously pointing out security concerns.

175. Several surveys have shown that skis purchased by competitive skiers are the biggest influence on customers buying

skis. Thus, all the innovation made by manufacturers in skis is limited by what is stipulated by the committee of the

winter olympics, the rule-setter for ski competitors, and any innovation, which will be deemed unfair by the

committee, is avoided.

Which of the following is an assumption made by the ski manufacturers?

A) Low-end skis that require no innovation are not popular at al.

B) Competitive skiers do not buy skis that violate the Winter Olympics committee stipulations, even when they are

skiing out of competititons.

C) The winter olympics committee does not support innovation.

D) Skis are bought by people of all ages.

E) Innovations in skis are not driven by ski manufacturers but by designers.

176. After our extensive inquiry into the theft of Jou-Mei’s pudding pop and ice cream sandwich, we have determined that

either Aingeru or Iskandar is responsible for the theft, as they were the only two people to have access to the fridge

between Jou-Mei’s deposit of the desserts and their unauthorized removal. A pudding pop was found in Aingeru’s

cubicle, leading us to suspect him of the theft, but after a witness came forward to accuse Iskandar, Iskandar has

admitted to taking the ice cream sandwich. As a result, we must apologize to Aingeru for wrongly accusing him of the

theft of the pudding pop.

Which of the following is an assumption on which this argument depends?

(A) Aingeru will accept the apology.

(B) Iskandar has a history of unauthorized food removal.

(C) Jou-Mei planned to eat both the desserts in question

(D) Aingeru actually consumed a fruit popsicle he had brought from home.

(E) Whoever took the pudding pop also took the ice cream sandwich.

177. People who have a college degree tend to live longer than those who do not have a college degree. This is true for

both men and women, and it is true across many different ethnicities and in many different geographical regions. This

phenomenon is easily explained because it has been proven that people who have attended college have lifestyles and

habits that are well known to increase lifespan: they eat balanced meals, they exercise often, they consider

themselves happy, and they tend to have strong relationships.

The explanation given in the argument above assumes which of the following?

A. The lifestyles and habits listed increase the lifespan of most people among different ethnicities and in different

Page 64: Critical Reasoning Questions - CATKing Educare · Critical Reasoning Questions . Borivali | Andheri |Powai & Pune Contact : 9029077114/ 8097607504 / 9619160261 / 8149774534 1. Homeowners

Borivali | Andheri |Powai & Pune

Contact : 9029077114/ 8097607504 / 9619160261 / 8149774534

www.catking.in

geographical regions.

B. There are other reasons that explain why someone with a college degree tends to have a longer lifespan.

C. All of the people who attend college finish and get their degree.

D. People without a college degree do not have similar lifestyles and habits to those with a college degree.

E. There are other factors that contribute to a longer lifespan than the ones listed.

178. Many employees of GEOCOM, a company established five years ago, have service vehicles. Since all employees of the

quality control department (QCD) live within ten miles of the company’s campus, it can be concluded that none of the

employees of the quality control department has a service vehicle provided by the company.

The conclusion drawn above depends on which of the following assumptions?

(A)No employee who lives within ten miles of the company’s campus has a service vehicle.

(B)None of the quality control department employees lives farther than fifteen miles from the company’s campus.

(C)Some employees of the marketing department live within ten miles of the company’s campus and have service

vehicles.

(D)All employees who live within ten miles from the company work for the quality control department.

(E)All employees who don’t have a service car are employees of the quality control department.

179. Editor: We use the computer to check the length of our articles, but surprisingly there is good reason to believe that

the word count it provides is inaccurate. Several times when an article's words were carefully counted by our most

reliable copy editor, the resulting count differed from the count the computer gave

The editor's reasoning relies on which of the following assumptions?

A) the criteria that the computer uses in determining what constitutes a single word differ from the criteria that the

copy editor uses

B) the inaccuracy of the computer's word count does not result from a malfunction of the computer itself

C) it would be possible to modify the computer so that it counted words more accurately

D) a careful count by the copy editor is unlikely to be less accurate than the computer's count

E) the accuracy of the computer's word count is not dependent on the length of the article that it is measuring

180. Chema is a factory that creates plastics and plastic related products for the chemical and pharmaceutical industries.

It has two options to reduce harmful emissions from its factory - installing scrubbers in its smokestacks and switching

to cleaner-burning fuel. Since scrubbers can reduce more harmful emissions than cleaner-burning fuels can, Chema is

doing the most it can do to reduce harmful emissions from its factory by installing scrubbers.

Which of the following is an assumption on which the argument depends?

(A) Switching to cleaner-burning fuel will not be more expensive than installing scrubbers.

(B) Chema is not necessarily committed to reducing harmful emissions from its factories.

(C) Harmful emissions from Chema's factories cannot be reduced more by using both methods together than by the

installation of scrubbers alone.

(D) Aside from harmful emissions from the smokestacks of its factories, the activities of Chema do not cause

significant air pollution.

(E) Chema can choose from various kinds of scrubbers, some of which are more effective than others.

Page 65: Critical Reasoning Questions - CATKing Educare · Critical Reasoning Questions . Borivali | Andheri |Powai & Pune Contact : 9029077114/ 8097607504 / 9619160261 / 8149774534 1. Homeowners

Borivali | Andheri |Powai & Pune

Contact : 9029077114/ 8097607504 / 9619160261 / 8149774534

www.catking.in

181. Residents of Milatia are known for their longevity. Nutritionists maintain that the Milatians can attribute their

increased lifespans to their diets. In addition to consuming a diet full of leafy greens, they also have a low intake of

saturated fats, which have been implicated in heart disease and atherosclerosis. Therefore, if one wants to have

increased longevity, he or she should follow a Milatia based diet.

Which one of the following is an assumption on which the argument depends?

A. Other aspects of the Militians’ lifestyle do not affect the observed trend of longevity in Milatia.

B. Adopting another people’s eating habits will, in of itself, not confer the same advantages, unless a person

incorporates exercise into his or her life.

C. The Milatian lifespan has a relatively uniform distribution, with very few dying young from natural causes.

D. Milatians are the only people in whom there is a perceived link between diet and longevity.

E. All Milatians are known to have a lifespans that are above average.

182. Civil trials often involve great complexities that are beyond the capacities of jurors to understand. As a result, jurors'

decisions in such trials are frequently incorrect. Justice would therefore be better served if the more complex trials

were decided by judges rather than juries.

The argument above depends on which of the following assumptions?

A. A majority of civil trials involve complexities that jurors are not capable of understanding.

B. The judges who would decide complex civil trials would be better able to understand the complexities of those

trials than jurors are.

C. The judges who would preside over civil trials would disallow the most complex sorts of evidence from being

introduced into those trials.

D. Jurors' decisions are frequently incorrect even in those civil trials that do not involve great complexities.

E. The sole reason in favor of having juries decide civil trials is the supposition that their decisions will almost always

be correct.

183. Jonathan: Have you heard about this new proposed development project? They want to build a 950-unit apartment

complex along the waterfront. That means that over 1,000 additional vehicles will be passing through the main

downtown intersection each day, which will increase

fumes and congestion to unacceptable levels.

Daria: Is that really such a large increase? When I'm sitting at the downtown coffee shop, I see at least 50 vehicles go

through that intersection every time the light changes. Assuming the light changes every three minutes, that's 1,000

vehicles passing through every hour.

Daria's argument relies on the assumption that

(A) changes in the traffic light patterns could mitigate the effect of the development

(B) the traffic at the time of day Daria sits at the coffee shop represents a typical level

(C) residents of apartment complexes usually own at least one car per apartment

(D) Jonathan's understanding of the impact of fumes and congestion on the quality of life

downtown is reasonably accurate

(E) large numbers are useful talking points for political opponents of development because

they are easily misinterpreted

Page 66: Critical Reasoning Questions - CATKing Educare · Critical Reasoning Questions . Borivali | Andheri |Powai & Pune Contact : 9029077114/ 8097607504 / 9619160261 / 8149774534 1. Homeowners

Borivali | Andheri |Powai & Pune

Contact : 9029077114/ 8097607504 / 9619160261 / 8149774534

www.catking.in

184. Crab fishing is not considered a sport by many laypeople. But because crab cages are extremely heavy, crab

fishermen must be very strong to do their work. For this reason, crab fishing is a sport.

Which of the following is an assumption that supports drawing the conclusion above for from the reason given for that

conclusion?

A) If an activity is a sport then it requires strength.

B) Crab fishing requires more strength than other types of fishing.

C) Crab fishermen have to do outside exercise to maintain their ability to fish.

D) Crab fishermen consider crab fishing a sport.

E) If an activity requires strength then it is a sport.

185. According to the new office smoking regulations, only employees who have enclosed offices may smoke at their

desks. Virtually all employees with enclosed offices are at the professional level, and virtually all secretarial

employees lack enclosed offices. Therefore, secretaries who smoke should be offered enclosed offices.

Which of the following is an assumption that enables the conclusion above to be properly drawn?

A) Employees at the professional level who do not smoke should keep their enclosed offices.

B) Employees with enclosed offices should not smoke at their desks, even though the new regulations permit them to

do so.

C) Employees at the secretarial level should be allowed to smoke at their desks, even if they do not have enclosed

offices.

D) The smoking regulations should allow all employees who smoke an equal opportunity to do so, regardless of an

employees job level.

E) The smoking regulations should provide equal protection from any hazards associated with smoking to all employees

who do not smoke.

186. Mayor: the commuters from the nearby suburb of Coldstream perennially complain of a difficult commute to our

downtown region. To address these concerns, we should install high-speed toll collector on the interstate between

here and Coldstream, thus relieving any traffic snarls caused by the hand-operated toll booths.

Civil engineer: Over 95% of commuters from Coldstream take local roads, not the interstate, to our downtown region.

The civil engineer uses which of the following techniques in responding to the mayor?

(A) pointing out that the premise could lead to an opposing conclusion

(B) questioning whether the methods recommended would work in practice

(C) citing evidence that calls into question the assumption of the argument

(D) suggesting, by analogy, that the argument might not support the conclusion

(E) presenting evidence that the proposed solution would have damaging unforeseen consequences

187. Bonuses at DSR Industries cannot be awarded unless profits exceed a ten percent return on stock- holders’

investments in the company. Higher profits mean higher bonuses. Therefore, bonuses in a year of general economic

recession will be considerably lower than bonuses in a year of peak profits at DSR.

The conclusion above depends on the assumption that

(A) the firm will have relatively low profits in recession years

Page 67: Critical Reasoning Questions - CATKing Educare · Critical Reasoning Questions . Borivali | Andheri |Powai & Pune Contact : 9029077114/ 8097607504 / 9619160261 / 8149774534 1. Homeowners

Borivali | Andheri |Powai & Pune

Contact : 9029077114/ 8097607504 / 9619160261 / 8149774534

www.catking.in

(B) the amount represented by a ten percent return on stockholders’ investments in the company will increase from

year to year

(C) profits rarely exceed a ten percent return on stockholders’ investments in the company

(D) profits in excess of a ten percent return on stockholders’ investments in the company are all distributed in the

form of bonuses

(E) bonuses at DSR never drop to zero

188. According to the new office smoking regulations, only employees who have enclosed office may smoke at their desks.

Virtually all employees with enclosed offices are at the professional level, and virtually all secretarial employees lack

enclosed offices. Therefore, secretaries who smoke should be offered enclosed offices.

Which of the following is an assumption that enables the conclusion above to be properly drawn?

(A) Employees at the professional level who do not smoke should keep their enclosed offices.

(B) Employees with enclosed offices should not smoke at their desks, even though the new regulations permit them to

do so.

(C) Employees at the secretarial level should be allowed to smoke at their desks, even if they do not have enclosed

offices.

(D) The smoking regulations should allow all employees who smoke an equal opportunity to do so, regardless of an

employee’s job level.

(E) The smoking regulations should provide equal protection from any hazards associated with smoking to all

employees who do not smoke.

189. Better-Vision-Maker is a liquid that enhances the visual ability of a person with average eyesight by almost 500 times.

The drug is intended for use by police, rescue, and medical service professionals. Research shows that a change in

visual ability can alter the dynamics of the brain so that the new functionality can be adapted to. More testing into

the side-effects of the drug should be performed before it is distributed for use.

The author's conclusion relies on which of the following assumptions?

A) The shift in the brain's dynamics caused by transition from one visual state to another may have an unfavorable

effect.

B) Testing can result in the finding of additional uses for the drug which will then increase its market value.

C) To handle the extreme challenges presented to them, police, rescue, and medical services need the use of any

practical tool available to them.

D) The brain adapts to increased visual ability in a similar way to an increased ability of another of the body's senses.

E) Enhancing the eyesight of the professionals mentioned will not actually result in an increase of their performance.

190. The rate of unemployment in Country X is higher than is the rate of unemployment in Country Y. Therefore, the

number of unemployed people in Country X is greater than the number of unemployed people in Country Y is.

Which of the following is an assumption underlying the above conclusion?

A) The unemployment rate in Country X is 15%.

B) The population of Country Y is more skilled than the population of Country X.

C) The number of people in Country X is the same as the number of people in Country Y.

D) The size of Country X is the same as that of Country Y.

E) In both countries, the ratio of the number of men to the number of women is the same.

Page 68: Critical Reasoning Questions - CATKing Educare · Critical Reasoning Questions . Borivali | Andheri |Powai & Pune Contact : 9029077114/ 8097607504 / 9619160261 / 8149774534 1. Homeowners

Borivali | Andheri |Powai & Pune

Contact : 9029077114/ 8097607504 / 9619160261 / 8149774534

www.catking.in

191. Having studied the mystic art of shamanism for seven years, Damian noticed that when the Chant of Rain was recited

by a group of shamans during the night of a full moon, the place mentioned in the chant would get rainfall within a

week. Damian thus concluded that it would be impossible to summon rain using the chant during the three weeks

before the full moon.

Damian's conclusion relies on which of the following assumptions?

A) Shamans only chant their chants on moonlit nights.

B) It only rains after the recitation of the Chant of Rain.

C) The Chant of Rain is only effective when recited under a full moon.

D) The shamans' ability to control weather has not been scientifically proven yet.

E) The chant is ineffective unless a group of experienced shamans recites it.

192. Although there is no record of poet Edmund Spenser's parentage, we do know that as a youth Spenser attended the

Merchant Tailors' School in London for a period between 1560 and 1570. Records from this time indicate that the

Merchant Tailors' Guild then had only three members named Spenser: Robert Spenser, listed as a gentleman; Nicholas

Spenser, elected the Guild's Warden in 1568; and John Spenser, listed as a "journeyman cloth-maker." Of these, the

last was likely the least affluent of the three—and most likely Edmund's father, since school accounting records list

Edmund as a scholar who attended the school at a reduced fee.

Which of the following is an assumption on which the argument depends?

(A) Anybody in sixteenth century London who made clothing professionally would have had to be a member of the

Merchant Tailors' Guild.

(B) The fact that Edmund Spenser attended the Merchant Tailors' School did not necessarily mean that he planned to

become a tailor.

(C) No member of the Guild could become Guild warden in sixteenth century London unless he was a gentleman.

(D) Most of those whose fathers were members of the Merchant Tailors' Guild were students at the Merchant Tailors'

School.

(E) The Merchant Tailors' School did not reduce its fees for the children of the more affluent Guild members.

193. The wedding gown industry in Country X has known little change in fashion over the past twenty years. The only

brides interested in new fashions are the ones who have spent a large portion of their lives abroad. Therefore, the

change in fashion in wedding gown design in Country X is limited to what is considered fashionable in foreign wedding

gown fashion shows.

The conclusion of the argument depends on which of the following assumptions?

A) Brides who have spent a large portion of their lives outside country X have usually developed their tastes for

wedding gowns based on wedding gown fashion shows they have seen there.

B) Women who have spent a long time abroad are not up to date about the fashion currently in vogue in their own

countries.

C) The ongoing stagnation in the wedding gown fashion in Country X results mainly from a lack of local fashion shows.

D) The slow pace of progress in the fashion of wedding gowns in Country X is closely related to the pace of progress in

fashion in general throughout the country.

Page 69: Critical Reasoning Questions - CATKing Educare · Critical Reasoning Questions . Borivali | Andheri |Powai & Pune Contact : 9029077114/ 8097607504 / 9619160261 / 8149774534 1. Homeowners

Borivali | Andheri |Powai & Pune

Contact : 9029077114/ 8097607504 / 9619160261 / 8149774534

www.catking.in

E) Due to tough economical conditions, local wedding dress designers cannot survive financially, so they relocate to

foreign countries.

194. Cognitive psychologist: The argument against IQ as a hereditary trait that is fixed throughout one’s lifetime ignores

the evidence of eye orientation. That is, there is a high correlation between the speed in which a person’s eyes orient

towards a stimulus and that person’s IQ. Specifically, an experiment measured the number of milliseconds subjects

required to orient their eyes to the where on a large screen a word was flashed. The study found that the more rapid

the response the higher that person’s IQ.

Which of the following is an assumption the cognitive psychologist makes?

A) The speed in which a person orients his or her eyes towards a stimulus is a skill that cannot be modified by

experience.

B) The ability of scientists to measure a person’s IQ depends on whether that person is literate.

C) Some subjects who have a documented high IQ orient towards a stimulus slower than the average.

D) The screen used in the experiment was so large that subjects had to shift their bodies in order to read the words

presented on the screen.

E) The speed at which one can orient one’s eye to a stimulus has also been highly correlated with overall health.

195. In the past century formerly consensual perceptions of the human palate have changed significantly. Man's culinary

likes and dislikes have proved to be less culturally ingrained in society than was once thought and dishes foreign to

many cultures have proliferated in these cultures across the globe in the last 100 years. A company marketing

packaged goods from Country X has deduced that it will be profitable to market its packaged goods of Country X's

cuisine to Country Y.

The company's deduction must rely on all the following assumptions EXCEPT:

A) Country Y will accept packaged goods.

B) Country Y will enjoy the specific foods of country X.

C) There is a way to transfer packaged goods from Country X to Country Y.

D) The company is the first from Country X to export packaged goods to Country Y.

E) The price the citizens of Country Y are willing to pay for the company's packaged goods exceeds the price of

research, production, and shipping the company must invest to sell its goods in Country Y.

196. Bamboo is slowly being recognized by a growing number of industries as a versatile and remarkable material. With

sources of wood becoming scarcer by the day, bamboo provides an ecological and sustainable substitute, especially

since it is the fastest growing plant on earth. Used in Asian countries for thousands of years, bamboo can provide

structural solutions for the production of tools, buildings, home utensils, furniture, and lighting. It can be said,

therefore, that if bamboo replaced the wood used in the manufacturing of these products, deforestation would no

longer be an ecological threat.

Which of the following is an assumption underlying the conclusion?

A) Replacing wood with bamboo to manufacture the products in the argument would be an uncomplicated procedure.

B) The rapid reduction of forest acreage is caused by using forest wood for the manufacturing of the products

mentioned.

Page 70: Critical Reasoning Questions - CATKing Educare · Critical Reasoning Questions . Borivali | Andheri |Powai & Pune Contact : 9029077114/ 8097607504 / 9619160261 / 8149774534 1. Homeowners

Borivali | Andheri |Powai & Pune

Contact : 9029077114/ 8097607504 / 9619160261 / 8149774534

www.catking.in

C) Bamboo is not more expensive than wood.

D) A manufacturing process that uses one material will differ to one that uses another.

E) Deforestation is an ecological threat that receives a high amount of public exposure.

197. Toddlers are watching television more than twice as much as they did two decades ago. A recent research showed

that because of this, toddlers today develop their motor skills slower than children their age in the compared period.

Therefore, toddlers who spend most of their time in front of the TV have less developed motor skills than those who

spend the majority of their time in other occupations.

The argument depends on which of the following assumptions?

A) Television can be used to enhance many skills of children aged 1-3.

B) Overall, watching television has a negative effect on young children.

C) Toddlers should be encouraged to play outdoors more.

D) Toddlers who watch less television are engaged in activities which develop their motor skills.

E) Only the physical removal of the television set from homes with young children will enable them to develop their

motor skills more quickly.

198. A new center celebrating all religions is to be built in Dawn City. Since statues depicting human beings are

considered blasphemous in certain religions, the architect of the new center avoided including statues of men or

women in her plans for the center so that no-one is offended.

Which of the following is an assumption made by the architect of the center in Dawn City

A) All statues may be found objectionable by some people.

B) Not only statues of men but also statues of women may be found to be objectionable by some people.

C) Statues of human beings are not essential to the worship of any religion.

D) The inappropriateness of statues is discussed at length in many scriptures.

E) Some religions incorporate statues of animals in their ritual.

199. Consumer advocate: Ephedra is a naturally occurring compound that some people use as a weight-loss aid. Recently,

the government prohibited the sale of dietary supplements containing ephedra on the grounds that ephedra has been

shown to have grave side effects. This prohibition is unreasonable. Echinacea is another natural compound that has

been shown to have side effects, yet echinacea is widely available at health food stores

The consumer advocate's argument depends on which of the following assumptions?

(A) Before the prohibition, ephedra had been available in health food stores

(B) All natural compounds are safe for human consumption.

(C) The side effects of echinacea and ephedra are comparably serious.

(D) The government should not hav the authority to prohibit natural compounds

(E) It is unreasonable to protect the health of the public.

200. Despite the fact that many professional writers consider travel writing a lesser form of journalism, it is in fact a

legitimate journalistic enterprise, since it employ s classical journalism techniques such as detailed research into the

history of a given locale and extensive interviews with local residents.

The argument above depends on which one of the following assumptions?

Page 71: Critical Reasoning Questions - CATKing Educare · Critical Reasoning Questions . Borivali | Andheri |Powai & Pune Contact : 9029077114/ 8097607504 / 9619160261 / 8149774534 1. Homeowners

Borivali | Andheri |Powai & Pune

Contact : 9029077114/ 8097607504 / 9619160261 / 8149774534

www.catking.in

(A) If a literary work is crafted via extensive interviews of noteworthy subjects it should be viewed as legitimate.

(B) Since travel writing follows the methods of traditional journalism, it will produce intriguing material for readers.

(C) Any writing that does not employ classical techniques is a lesser form of journalism.

(D) If a literary pursuit involves classical journalism techniques, then it should be considered a legitimate journalistic

enterprise.

(E) The interview process used by travel writers can provide further information about the history of a region.

Answer Key

1. A 41. D 81. A 121. A 161. A 2. D 42. D 82. E 122. B 162. C 3. D 43. D 83. D 123. C 163. C 4. B 44. D 84. D 124. A 164. B 5. E 45. E 85. D 125. A 165. A 6. A 46. C 86. A 126. B 166 C 7. E 47. C 87. C 127. A 167. A 8. C 48. E 88. E 128. C 168. A 9. B 49. D 89. E 129. E 169. C

10. D 50. D 90. C 130. E 170. C 11. C 51. C 91. C 131. B 171. E 12. C 52. B 92. B 132. E 172. A 13. E 53. C 93. E 133. A 173. A 14. E 54. E 94. D 134. D 174. C 15. B 55. E 95. A 135. D 175. B 16. A 56. A 96. C 136. E 176. E 17. E 57. B 97. C 137. C 177. D 18. A 58. C 98. C 138. E 178. A 19. D 59. D 99. E 139. C 179. D 20. D 60. D 100. C 140. C 180. C 21. B 61. D 101. E 141. C 181. A 22. A 62. C 102. E 142. E 182. B 23. D 63. C 103. C 143. D 183. B 24. A 64. D 104. B 144. D 184. E

Page 72: Critical Reasoning Questions - CATKing Educare · Critical Reasoning Questions . Borivali | Andheri |Powai & Pune Contact : 9029077114/ 8097607504 / 9619160261 / 8149774534 1. Homeowners

Borivali | Andheri |Powai & Pune

Contact : 9029077114/ 8097607504 / 9619160261 / 8149774534

www.catking.in

25. C 65. B 105. D 145. B 185. D 26. C 66. B 106. A 146. C 186. C 27. E 67. B 107. A 147. C 187. A 28. C 68. E 108. A 148. B 188. D 29. D 69. C 109. B 149. C 189. A 30. E 70. B 110. C 150. C 190. C 31. C 71. D 111. E 151. C 191. C 32. F 72. D 112. E 152. E 192. E 33. E 73. B 113. D 153. E 193. A 34. C 74. E 114. B 154. E 194. A 35. D 75. B 115. E 155. C 195. D 36. C 76. A 116. A 156. E 196. B 37. C 77. A 117. A 157. C 197. D 38. D 78. B 118. C 158. C 198. C 39. D 79. C 119. B 159. C 199. C 40. A 80. E 120. A 160. C 200. D